CARDIAC - 4th semester- URDEN

Réussis tes devoirs et examens dès maintenant avec Quizwiz!

A patient recovering from a myocardial infarction (MI) notifies the nurse that he is having chest pain. Upon listening to the patient's heart sounds, the nurse hears a grating sound that is present during both systole and diastole. The nurse suspects that the patient's chest pain is most likely caused by which condition? A Another MI B Inflammation of the pericardium C Papillary muscle rupture D Ventricular septal rupture

(EVOLVE)

Which cardiac medication is a synthetic catecholamine with predominantly beta1-adrenergic effects? It also produces some beta2 stimulation, resulting in a mild vasodilation. Dobutamine Dopamine Levophed Phenylephrine

A Dobutamine is a synthetic catecholamine with predominantly beta(EVOLVE)

A patient admitted with sepsis has a pulmonary artery catheter placed that also measures SvO2. During a routine assessment, the nurse notes that the patient's SvO2 is 56%. What does this finding indicate? A Oxygen supply is not equal to demand. B Oxygen supply is equal to demand. C Oxygen saturation is not accurate. D Oxygen is not being extracted by the tissues.

A (EVOLVE)

In an effort to improve cardiac output, a positive inotropic drug is given to: A increase the contractility of the heart. B increase the heart rate. C increase the conduction velocity. D increase the automaticity of the heart.

A (EVOLVE)

Pulse pressure is the difference between systolic pressure and diastolic pressure. What will happen to the patient's pulse pressure if the patient becomes volume depleted? A Narrow B Widen C Remain unchanged D Increase

A (EVOLVE)

Which structure stores calcium ions for release and use after depolarization? A Sarcoplasmic reticulum B Sarcomere C Sarcolemma D Transverse tubules

A (EVOLVE)

Which hemodynamic changes are associated with mitral stenosis? A Elevated left atrial pressure, pulmonary artery occlusive pressure, and pulmonary artery diastolic pressure; normal left ventricular end-diastolic pressure B Elevated left atrial pressure, pulmonary artery occlusive pressure, and left ventricular end-diastolic pressure; normal pulmonary artery diastolic pressure C Elevated pulmonary artery occlusive pressure, pulmonary artery diastolic pressure, and left ventricular end-diastolic pressure; normal left atrial pressure D Elevated left atrial pressure, pulmonary artery diastolic pressure, and left ventricular end-diastolic pressure; normal pulmonary artery occlusive pressure

A (EVOLVE) In mitral valve stenosis, left atrial pressure and pulmonary artery occlusive pressure are increased and cause pulmonary congestion; however, these elevated values do not reflect the left ventricular end-diastolic pressure (LVEDP) because a stenotic mitral valve decreases normal blood flow from the left atrium to the left ventricle, decreasing left ventricular preload and consequently lowering LVEDP. The other options do not accurately describe the hemodynamic effects of mitral stenosis.

An inferior myocardial infarction (MI) occurs with an occlusion to the right coronary artery. In which leads would the electrocardiograph (ECG) changes be evident? II, III, and aVF I, aVL, V5, and V6 V1 to V4 V1 to V2

A (EVOLVE) Inferior wall infarction occurs with occlusion of the right coronary artery. This infarction manifests by electrocardiographic (ECG) changes in leads II, III, and aV

A patient has a diastolic murmur located at the fifth intercostal space (ICS) midclavicular line (MCL). The nurse knows this is indicative of what disorder? A Mitral stenosis B Tricuspid regurgitation C Aortic regurgitation D Pulmonic stenosis

A (EVOLVE) Mitral stenosis produces a diastolic murmur that is heard at the fifth intercostal space (ICS) midclavicular line (MCL). Tricuspid regurgitation and pulmonic stenosis are systolic murmurs. Aortic regurgitation is a diastolic murmur that is heard at the second ICS right sternal border.

A patient is admitted with syncope, exertional dyspnea, and a systolic murmur. Cardiac catheterization reveals significantly increased left ventricular end-diastolic pressure (LVEDP). The nurse suspects the patient may be experiencing which problem? Aortic stenosis Mitral stenosis Tricuspid stenosis Pulmonary regurgitation

A Symptoms of aortic stenosis include syncope, exertional dyspnea, increased left ventricular end-diastolic pressure (LVEDP), and systolic murmur. Mitral and tricuspid stenoses are associated with a diastolic murmur as is pulmonary regurgitation.Cognitive Level: EvaluatingNursing Process: AssessmentClient Need: Physiological Integrity: Physiological Adaptation (EVOLVE)

Which patients are candidates for fibrinolytic therapy? Select all that apply. A A 38-year-old patient with chest pain for 8 hours and ST elevation in leads V1 to V4 B A 47-year-old patient with bundle branch block (BBB) and ST elevation in leads II, III, and aVF and new onset left BBB C A 54-year-old patient who had a surgical repair of gastric ulcer 1 week ago, with chest pain for 10 hours D A 64-year-old patient with chest pain for 6 hours after a motor vehicle crash in which the airbag was deployed E A 49-year-old patient with chest pain for 14 hours and a history of BBB

A B Patients with a history of trauma, surgery, or stroke are not candidates for fibrinolytics, nor are patients with chest pain for longer than 12 hours. A 38-year-old patient with chest pain for 8 hours and ST elevation in leads V1 to V4 and a 47-year-old patient with bundle branch block (BBB) and ST elevation in leads II, III, and aVF and new onset left BBB are both candidates for fibrinolytics.Cognitive Level: ApplyingNursing Process: PlanningClient Need: Physiological Integrity: Pharmacological and Parenteral Therapies (EVOLVE)

The nurse is explaining the function of the atrioventricular (AV) node to the student nurse. Which statement is true? Select all that apply. A The conduction time of impulses from the sinoatrial (SA) to the AV node allows the ventricles to fill with blood during diastole. B The AV node prevents rapid heart rates from destabilizing the heart. C The AV node provides a backup pacemaker if the SA node fails. D The AV node sends impulses forward only. E The AV is located on the left side of the interatrial septum.

A B C (EVOLVE)

The nurse is leveling and rezeroing the patient's hemodynamic line upon returning from the radiology department. The patient asks the nurse, "Why are you doing that?" What is an appropriate response? Select all that apply. A "Leveling the transducer above or below the reference point on your body can result in erroneous readings." B "Zeroing removes the effects of atmospheric pressure on the readings." C "If we use the same reference point, we will obtain consistent measurements." D "It is important to line up the reference point to the left side of the top of your heart." E "This is something we do periodically."

A B C D (EVOLVE) Leveling the transducer above or below the reference point on your body can result in erroneous readings; zeroing removes the effects of atmospheric pressure on the readings; if we use the same reference point, we will obtain consistent measurements; and it is important to line up the reference point to the left side of the top of your heart are all correct and appropriate statements to share with a patient. This is something we do periodically is an incomplete and inappropriate response to a patient's question.Cognitive Level: ApplyingNursing Process: ImplementationClient Need: Physiological Integrity: Reduction of Risk Potential

Cardiac output is the amount of blood ejected from the heart in 1 minute. If a patient's heart rate (HR) is 72 beats/min and the stroke volume (SV) is 70 mL/beat, what is the patient's cardiac output? A 2 L/min B 5.04 L/min C 142 L/min D 120 L/min

B (EVOLVE)

The nurse is assessing a newly admitted patient. Which finding would indicate a chronic cardiac history? Select all that apply. A The patient states he has a "beer belly." B The patient's fingernails are thickened, yellow, brittle, and cracked. C The patient has a visible pulse at the fifth intercostal space just lateral to the midclavicular line. D The patient has a capillary refill time of 2 seconds. E The patient has a large, firm bulge noted on the left upper chest.

A C E (EVOLVE) Abdominal adipose is often related to cardiac problems (an "apple-shaped" body). A visible point of maximal impulse (PMI) is associated with a cardiac history. A large, firm bulge on the left upper chest is a sign of an implanted pacemaker. Thickened, yellow, brittle, and cracked fingernails are a sign of nail infection, and a capillary refill time of 2 seconds is normal.

20. A patient is being monitored by continuous ECG after placement of a transvenous pacemaker. "Loss of capture" is seen on the ECG. Which nursing intervention may correct this situation? a. Position the patient on the left side. c. Increase the rate as ordered. b. Decrease the milliamperes as ordered. d. Monitor the patient in a different lead.

ANS: A "Loss of capture" most often can be attributed either to displacement of the pacing electrode or to an increase in threshold as a result of drugs, metabolic disorders, electrolyte imbalances, or fibrosis or myocardial ischemia at the site of electrode placement. In many cases, increasing the output (mA) may elicit capture. For transvenous leads, repositioning the patient to the left side may improve lead contact and restore capture.

62. A patient's arterial line waveform has become damped. The nurse should a. check for kinks, blood, and air bubbles in the tubing. b. prepare for a normal saline fluid challenge for hypotension. c. discontinue the arterial line. d. check the patient's lung sounds.

ANS: A A damped waveform occurs when communication from the artery to the transducer is interrupted and produces false values on the monitor and oscilloscope. Damping is caused by a fibrin "sleeve" that partially occludes the tip of the catheter, by kinks in the catheter or tubing, or by air bubbles in the system.

12. Adenosine (Adenocard) is an antidysrhythmic agent given primarily to a. convert supraventricular tachycardias. b. act as a second-line drug for premature ventricular contractions (PVCs). c. treat AV blocks. d. coarsen ventricular fibrillation so that defibrillation is effective.

ANS: A Adenosine occurs endogenously in the body as a building block of adenosine triphosphate (ATP). Given in intravenous boluses, adenosine slows conduction through the atrioventricular (AV) node, causing transient AV block. It is used clinically to convert supraventricular tachycardias and to facilitate the differential diagnosis of rapid dysrhythmias.

20. A sudden increase in left atrial pressure, acute pulmonary edema, and low cardiac output, caused by the ventricle contracting during systole, are all characteristics of a. acute mitral regurgitation. c. chronic mitral regurgitation. b. aortic insufficiency. d. pericardial friction rub.

ANS: A Acute mitral regurgitation occurs when the ventricle contracts during systole and a jet of blood is sent in a retrograde manner to the left atrium, causing a sudden increase in left atrial pressure, acute pulmonary edema, and low CO and leading to cardiogenic shock. Chronic mitral regurgitation is auscultated in the mitral area and occurs during systole. It is high pitched and blowing, although the pitch and intensity vary, depending on the degree of regurgitation. As mitral regurgitation progresses, the murmur radiates more widely. Aortic insufficiency is an incompetent aortic valve. If the valve cusps do not maintain this seal, the sound of blood flowing back into the left ventricle during diastole is heard as a decrescendo, high-pitched, blowing murmur. A pericardial friction rub is a sound that can occur within 2 to 7 days after a myocardial infarction. The friction rub results from pericardial inflammation (pericarditis). Classically, a pericardial friction rub is a grating or scratching sound that is both systolic and diastolic, corresponding to cardiac motion within the pericardial sac.

2. The abdominojugular reflux test determines the presence of a. right ventricular failure. c. liver failure. b. hypoxemia. d. pitting edema.

ANS: A The abdominojugular reflux sign can assist with the diagnosis of right ventricular failure. A positive abdominojugular reflux sign is an increase in the jugular venous pressure (CVP equivalent) of greater than 3 cm sustained for at least 15 seconds.

55. Which of the following statements made by a patient would indicate the need for further education before an electrophysiology procedure? a. "I need to take all my heart medications the morning of the procedure." b. "The doctor is going to make my heart beat wrong on purpose." c. "I will be awake but relaxed during the procedure." d. "I will be x-rayed during the procedure."

ANS: A All antidysrhythmic medications are discontinued several days before the study so that any ventricular dysrhythmias may be readily induced during the electrophysiology procedure (EPS). Anticoagulants, especially warfarin, are also stopped before EPS. Premedication is administered before the study to induce a relaxed state, and during the procedure, the patient is conscious but receives sedative agents (midazolam) at regular intervals.

17. Which of the following clinical manifestations is usually the first symptom of peripheral arterial disease (PAD)? a. Cramping when walking c. Pulmonary embolism b. Thrombophlebitis d. Cordlike veins

ANS: A Arterial occlusion obstructs blood flow to the distal extremity. The lack of blood flow produces ischemic muscle pain known as intermittent claudication. This cramping, aching pain while walking is often the first symptom of peripheral arterial occlusive disease. The pain is relieved by rest and may remain stable in occurrence and intensity for many years.

29. The current standard of care for PCI typically includes dual antiplatelet therapy with oral a. aspirin and Clopidogrel (Plavix). c. aspirin and Eptifibatide (Integrilin). b. aspirin and Abciximab (ReoPro). d. aspirin and Tirofiban (Aggrastat).

ANS: A Because platelet activation is a complex process involving multiple pathways, combination therapy with two or more agents has proven most effective. The current standard of care for percutaneous coronary intervention typically includes dual antiplatelet therapy with aspirin and a thienopyridine. These oral agents are administered before the procedure and continued at discharge. Abciximab, eptifibatide, and tirofiban are all intravenous antiplatelet agents.

5. A patient presents with atrial fibrillation, a heart rate of 156 beats/min, and a blood pressure of 124/76 mm Hg. The physician orders diltiazem, a calcium channel blocker, to be given slowly by intravenous push. Why did the physician choose this medication to treat this patient's atrial tachyarrhythmia? a. Diltiazem decreases the calcium influx into the AV nodal tissue and decreases the speed of impulse conduction. b. Diltiazem increases the calcium influx into the AV nodal tissue and decreases the speed of impulse conduction. c. Diltiazem decreases the calcium influx into the myocardial tissue and decreases the strength of heart contraction. d. Diltiazem increases the calcium influx into the myocardial tissue and decreases the strength of heart contraction.

ANS: A Calcium channel-blocking drugs, such as verapamil and diltiazem, inhibit the inward Ca++ current into pacemaker tissue, especially the AV node. For this reason, they are used therapeutically to slow the rate of atrial tachydysrhythmias and protect the ventricle from excessive atrial impulses.

1. Which of the following conditions is usually associated with clubbing? a. Central cyanosis c. Carbon monoxide poisoning b. Peripheral cyanosis d. Acute hypoxemia

ANS: A Clubbing in the nail bed is a sign associated with longstanding central cyanotic heart disease or pulmonary disease with hypoxemia. Peripheral cyanosis, a bluish discoloration of the nail bed, is seen more commonly. Peripheral cyanosis results from a reduction in the quantity of oxygen in the peripheral extremities from arterial disease or decreased cardiac output. Central cyanosis is a bluish discoloration of the tongue and sublingual area. Multiracial studies indicate that the tongue is the most sensitive site for observation of central cyanosis.

50. Which of the following has become the first-line hemodynamic assessment tool in the critical care unit? a. Echocardiogram c. Exercise stress test b. ECG d. 24-hour Holter monitor

ANS: A Echocardiography is quickly becoming a first-line hemodynamic assessment tool in critical care units. Echocardiography is used to detect structural heart abnormalities such as mitral valve stenosis and regurgitation, prolapse of mitral valve leaflets, aortic stenosis and insufficiency, hypertrophic cardiomyopathy, atrial septal defect, thoracic aortic dissection, cardiac tamponade, and pericardial effusion.

18. An echocardiogram reveals an ejection fraction of 55%. On the basis of this information, the patient's cardiac function is a. adequate. c. moderately decreased. b. mildly decreased. d. severely decreased.

ANS: A Ejection fraction is expressed as a percent, with normal being at least greater than 50%. An ejection fraction of less than 35% indicates poor ventricular function (as in cardiomyopathy), poor ventricular filling, obstruction to outflow (as in some valve stenosis conditions), or a combination of these.

13. Pulsus paradoxus may be seen on intra-arterial blood pressure monitoring when a. there is a decrease of more than 10 mm Hg in the arterial waveform before inhalation. b. there is a single, nonperfused beat. c. the waveform demonstrates tall, tented T waves. d. the pulse pressure is greater than 20 mm Hg on exhalation.

ANS: A Pulsus paradoxus is a decrease of more than 10 mm Hg in the arterial waveform that occurs during inhalation. It is caused by a fall in CO as a result of increased negative intrathoracic pressure during inhalation.

63. A 24-hour event monitor is commonly known as a. a Holter monitor. c. an external loop recorder. b. a transtelephonic monitor. d. a signal-averaged electrocardiogram.

ANS: A Holter monitors are the most widely used continuous recording systems. The monitor is carried by a shoulder strap or clipped to a belt or pocket for 24 hours and then is returned to the hospital or clinic for reading. Transtelephonic monitors are intermittent monitoring systems that are not attached to the patient all the time. These monitors consist of a small box, about 4 inches by 2 inches, with four metal electrodes on the bottom. The box is issued to the patient for a specific time, often 1 month, and the patient carries the box at all times. The external loop recorder records continuously but only keeps the most recent 4 minutes of electrocardiographic (ECG) activity on the memory loop. The memory loop in the device means the patient can press a button and intermittently record a specific event such as heart palpitations during or after the event has occurred. The signal-averaged ECG (SAECG) is used to identify late potentials within the QRS complex. These low-amplitude waveforms cannot be detected on a standard surface ECG but can be recorded as an SAECG.

26. The physician anticipates the CVC dwelling time to be 10 to 20 days. The nurse anticipates that the CVC will be placed in the a. SC vein. c. EJ vein. b. IJ vein. d. femoral vein.

ANS: A If the anticipated central venous catheter (CVC) dwelling time is prolonged more than 5 days, the subcutaneous (SC) site is preferred. The SC position has the lowest infection rate and produces the least patient discomfort from the catheter. The internal jugular (IJ) vein is the most frequently used access site for CVC insertion. Compared with the other thoracic veins, it is the easiest to canalize. If the IJ vein is not available, the external jugular (EJ) vein may be accessed, although blood flow is significantly higher in the IJ vein, making it the preferred site. This may be the reason why catheter-related infections are higher in the IJ than the SC position for indwelling catheters left in place for more than 4 days. The femoral vein is considered the easiest cannulation site because there are no curves in the insertion route. Because there is a higher rate of nosocomial infection with femoral catheters, this site is not recommended.

8. When assessing a patient with PVCs, the nurse knows that the ectopic beat is multifocal because it appears a. in various shapes in the same lead. c. to widen the QRS width. b. with increasing frequency. d. in a specific pattern in the same lead.

ANS: A If the ventricular ectopic beats are of various shapes in the same lead, they are multifocal. Multifocal ventricular ectopics are more serious than unifocal ventricular ectopics because they indicate a greater area of irritable myocardial tissue and are more likely to deteriorate into ventricular tachycardia or fibrillation.

43. New-onset atrial fibrillation can be serious for which of the following reasons? a. It increases the risk of stroke and pulmonary embolism from atrial clots. b. It increases the patient's risk of deep venous thrombosis. c. It may increase cardiac output to dangerous levels. d. It indicates that the patient is about to have an MI.

ANS: A In atrial fibrillation the atria do not contract normally; they quiver. This increases the chance of the blood clotting in the atria because of a lack of complete emptying of the atria. These clots can break free and cause embolic strokes and pulmonary emboli. Atrial fibrillation does not indicate impending myocardial infarction or an increased risk of deep venous thrombosis. Atrial fibrillation decreases cardiac output from the loss of atrial kick.

2. In caring for a postoperative cardiovascular patient, the nurse knows that the most frequent cause of a decreased cardiac output is a. reduced preload. c. increased contractility. b. increased afterload. d. bradycardia.

ANS: A In most patients, reduced preload is the cause of low postoperative cardiac output. To enhance preload, volume may be administered in the form of crystalloid, colloid, or packed red blood cells.

30. A physician orders removal of the central venous catheter (CVC) line. The patient has a diagnosis of heart failure with chronic obstructive pulmonary disease. The nurse would place the patient in what position for this procedure? a. Supine in bed c. Flat in bed b. Supine in a chair d. Reverse Trendelenburg position

ANS: A Recommended techniques to avoid air embolus during CVC removal include removing the catheter when the patient is supine in bed (not in a chair) and placing the patient flat or in the reverse Trendelenburg position if the patient's clinical condition permits this maneuver. Patients with heart failure, pulmonary disease, and neurologic conditions with raised intracranial pressure should not be placed flat.

11. In the acute phase after STEMI, fibrinolytic therapy is used in combination with heparin to recanalize the coronary artery. The initial heparin bolus is a. 60 units/kg maximum 5000 units. c. 25 units/kg maximum of 2500 units. b. 30 units/kg maximum 3000 units. d. 12 units/kg maximum of 1000 units.

ANS: A In the acute phase after ST segment elevation myocardial infarction, heparin is administered in combination with fibrinolytic therapy to recanalize (open) the coronary artery. For patients who will receive fibrinolytic therapy, an initial heparin bolus of 60 units/kg (maximum, 5000 units) is given intravenously followed by a continuous heparin drip at 12 units/kg/hr (maximum 1000 units/hr) to maintain an activated partial thromboplastin time between 50 and 70 seconds (1.5-2.0 times control).

20. A 66-year-old patient is admitted to the critical care unit with a diagnosis of acute inferior MI. A 12-lead electrocardiogram (ECG) is done to validate the area of infarction. For the above patient, which leads on the ECG would correlate with an inferior wall MI? a. II, III, aVF c. V2 to V4 b. V5 to V6, I, aVL d. V1 to V2

ANS: A Inferior infarctions are manifested by electrocardiographic (ECG) changes in leads II, III, and aVF. Lateral wall infarctions are manifested by ECG changes in leads V5 to V6, I, and aVL. Anterior wall infarctions are manifested by ECG changes in leads V2 to V4. Posterior wall infarctions are manifested by ECG changes in leads V1 to V2.

7. A reliable indicator of reperfusion after fibrinolytic therapy is a. dysrhythmias. c. elevated ST segments. b. Q waves. d. a rapid decrease in serum CK levels.

ANS: A Initially, when there is reperfusion, ischemic chest pain ceases abruptly as blood flow is restored. Another reliable indicator of reperfusion is the appearance of various "reperfusion" dysrhythmias. Premature ventricular contractions, bradycardias, heart block, ventricular tachycardia, and (rarely) ventricular fibrillation may occur.

11. An 82-year-old patient is admitted into the critical care unit with a diagnosis of left-sided heart failure related to mitral stenosis. Physical assessment findings reveal tachycardia with an S3 and a 3/6 systolic murmur. The grading of a murmur as a 3/6 refers to which of the following characteristics of murmurs? a. Intensity c. Timing b. Quality d. Pitch

ANS: A Intensity, or the "loudness," is graded on a scale of 1 to 6; the higher the number, the louder the murmur.

13. _____ are created by the turbulence of blood flow through a vessel caused by constriction of the blood pressure cuff. a. Korotkoff sounds c. Murmurs b. Pulse pressures d. Gallops

ANS: A Korotkoff sounds are the sounds created by turbulence of blood flow within a vessel caused by constriction of the blood pressure cuff. Pulse pressure describes the difference between systolic and diastolic values. The normal pulse pressure is 40 mm Hg. Abnormal heart sounds are known as the third heart sound (S3) and the fourth heart sound (S4); they are referred to as gallops when auscultated during an episode of tachycardia. Murmurs are produced by turbulent blood flow through the chambers of the heart, from forward flow through narrowed or irregular valve openings, or backward regurgitate flow through an incompetent valve.

4. One differentiating factor between stable angina and unstable angina is that stable angina a. responds predictably well to nitrates. c. has a low correlation to CAD. b. is not precipitated by activity. d. is a result of coronary artery spasm.

ANS: A Pain control is achieved by rest and by sublingual nitroglycerin within 5 minutes. Stable angina is the result of fixed lesions (blockages) of more than 75% of the coronary artery lumen. Ischemia and chest pain occur when myocardial demand from exertion exceeds the fixed blood oxygen supply.

4. The mechanism of dilation with percutaneous transluminal coronary angioplasty (PTCA) is a. stretching of the vessel wall, resulting in fracture of the plaque. b. anticoagulation after the completion of the procedure, enhancing dilation. c. plaque removal after balloon inflation. d. compression of plaque against the vessel wall.

ANS: A Percutaneous transluminal coronary angioplasty involves the use of a balloon-tipped catheter that, when advanced through an atherosclerotic lesion (atheroma), can be inflated intermittently for the purpose of dilating the stenotic area and improving blood flow through it. The high balloon-inflation pressure stretches the vessel wall, fractures the plaque, and enlarges the vessel lumen.

13. Medical management of hypertrophic cardiomyopathy includes a. beta-blockers. c. an intensive exercise regimen. b. positive inotropes. d. aortic valve replacement

ANS: A Pharmacologic management includes beta-blockers to decrease left ventricular workload, medications to control and prevent atrial and ventricular dysrhythmias, anticoagulation if atrial fibrillation or left ventricular thrombi are present, and finally drugs to manage heart failure.

26. The patient is 12 hours postoperative for a CABG. The patient's vital signs include: T 103° F, HR 112, RR 22, BP 134/78 mm Hg, and O2 sat 94% on 3L/NC. The nurse suspects that the patient has developed a. infection and notifies the physician immediately. b. infection, which is common postoperatively, and monitors the patient's condition. c. cardiac tamponade and notifies the physician immediately. d. delirium caused by the elevated temperature.

ANS: A Postoperative fever is fairly common after cardiopulmonary bypass. However, persistent temperature elevation to greater than 101° F (38.3° C) must be investigated. Sternal wound infections and infective endocarditis are the most devastating infectious complications, but leg wound infections, pneumonia, and urinary tract infections also can occur. A potentially lethal complication, cardiac tamponade may occur after surgery if blood accumulates in the mediastinal space, impairing the heart's ability to pump. Signs of tamponade include elevated and equalized filling pressures (e.g., central venous pressure, pulmonary artery diastolic pressure, pulmonary artery occlusion pressure), decreased cardiac output, decreased blood pressure, jugular venous distention, pulsus paradoxus, muffled heart sounds, sudden cessation of chest tube drainage, and a widened cardiac silhouette on radiographs. The risk of delirium is increased in cardiac surgery patients, especially elderly patients, and is associated with increased mortality rates and reduced quality of life and cognitive function. Nursing staff can play a critical role in the prevention and recognition of delirium.

65. Which of the following is most indicative of decreased left ventricular preload? a. Increased PAOP/PAWP c. Increased CVP b. Decreased PAOP/PAWP d. Decreased CVP

ANS: A Pulmonary artery occlusion pressure (PAOP) and pulmonary artery wedge pressure (PAWP) normally reflect the pressure in the left ventricle at the end of diastole. Left ventricular end-diastolic pressure is preload, and so an increase in preload will first increase the PAOP and PCWP measurements. Although central venous pressure increases in severe cases of increased preload, it can increase for other reasons. The other changes are not associated with increased preload.

6. An assessment finding of pulsus alternans may indicate evidence of a. left-sided heart failure. c. pulmonary embolism. b. jugular venous distention. d. myocardial ischemia.

ANS: A Pulsus alternans describes a regular pattern of pulse amplitude changes that alternate between stronger and weaker beats. This finding is suggestive of end-stage left ventricular heart failure.

6. One of the reasons that contributes to higher mortality rates from acute MI in women is: a. waiting longer to seek medical care. b. being younger when symptoms occur. c. risk factors associated with MI are more stronger in women than men. d. women have smaller hearts than men.

ANS: A Risk factors associated with acute myocardial infarction (MI) more strongly in women than in men include hypertension, diabetes mellitus, alcohol intake, and physical inactivity. Many reasons contribute to higher mortality rates from acute MI in women, and these include waiting longer to seek medical care, having smaller coronary arteries, being older when symptoms occur, and experiencing very different symptoms from those of men of the same age.

8. The function of the AV valves is to a. prevent backflow of blood into the atria during ventricular contraction. b. prevent blood regurgitation back into the ventricles. c. assist with blood flow to the lungs and aorta. d. contribute to ventricular filling by atrial kick.

ANS: A The AV valves are open during ventricular diastole (filling) and prevent backflow of blood into the atria during ventricular systole (contraction). Semilunar valves prevent the backflow of pulmonic and aortic blood back into the ventricles.

3. The purpose of the Allen test is to a. assess adequate blood flow through the ulnar artery. b. occlude the brachial artery and evaluate hypoxemia to the hand. c. test the patency of an internal graft. d. determine the size of needle to be used for puncture.

ANS: A The Allen test assesses the adequacy of blood flow to the hand through the ulnar artery.

60. The Allen test is used before radial arterial line placement to assess a. collateral circulation to the hand. c. neurologic function of the hand. b. patency of the radial artery. d. pain sensation at the insertion point.

ANS: A The Allen test involves occluding the radial or ulnar artery after blanching the hand. If the hand turns pink, then the nonoccluded artery provides enough circulation to the hand. If the hand remains blanched, then no collateral circulation exists, and that wrist should not be used for arterial line placement.

9. Clinical manifestations of right-sided heart failure include a. elevated central venous pressure and sacral edema. b. pulmonary congestion and jugular venous distention. c. hypertension and chest pain. d. liver tenderness and pulmonary edema.

ANS: A The common manifestations of right ventricular failure are the following: jugular venous distention, elevated central venous pressure, weakness, peripheral or sacral edema, hepatomegaly (enlarged liver), jaundice, and liver tenderness. Gastrointestinal symptoms include poor appetite, anorexia, nausea, and an uncomfortable feeling of fullness.

23. A patient's bedside ECG strips show the following changes: increased PR interval; increased QRS width; and tall, peaked T waves. Vital signs are T 98.2° F; HR 118 beats/min; BP 146/90 mm Hg; and RR 18 breaths/min. The patient is receiving the following medications: digoxin 0.125 mg PO every day; D51/2 normal saline with 40 mEq potassium chloride at 125 mL/hr; Cardizem at 30 mg PO q8h; and aldosterone at 300 mg PO q12h. The physician is notified of the ECG changes. What orders should the nurse expect to receive? a. Change IV fluid to D51/2 normal saline and draw blood chemistry. b. Give normal saline with 40 mEq of potassium chloride over a 6-hour period. c. Hold digoxin and draw serum digoxin level. d. Hold Cardizem and give 500 mL normal saline fluid challenge over a 2-hour period.

ANS: A The electrocardiographic (ECG) changes are most consistent with hyperkalemia. Removing the potassium from the intravenous line and drawing laboratory values to check the potassium level is the best choice with the least chance of further harm. Digoxin toxicity can be suspected related to the prolonged PR interval, but hyperkalemia explains all the ECG changes. The patient is not hypotensive or bradycardic, so holding the Cardizem is not indicated.

57. Zeroing the pressure transducer on hemodynamic monitoring equipment occurs when the displays reads a. 0. c. 600. b. 250. d. 760.

ANS: A The monitor is adjusted so that "0" is displayed, which equals atmospheric pressure. Atmospheric pressure is not zero; it is 760 mm Hg at sea level. Using zero to represent current atmospheric pressure provides a convenient baseline for hemodynamic measurement purposes.

6. A nurse is providing care to a patient on fibrinolytic therapy. Which of the following statements from the patient warrants further assessment and intervention by the critical care nurse? a. "My back is killing me!" b. "There is blood on my toothbrush!" c. "Look at the bruises on my arms!" d. "My arm is bleeding where my IV is!"

ANS: A The nurse must continually monitor for clinical manifestations of bleeding. Mild gingival bleeding and oozing around venipuncture sites are common and not causes for concern. However, severe lower back pain and ecchymoses are suggestive of retroperitoneal bleeding. If serious bleeding occurs, all fibrinolytic heparin therapies are discontinued, and volume expanders, coagulation factors, or both are administered.

17. A transvenous pacemaker is inserted through the right subclavian vein and threaded into the right ventricle. The pacemaker is placed on demand at a rate of 70. The pacing mode for this pacemaker is a. VVI. c. DDD. b. AOO. d. VAT.

ANS: A The original code is based on three categories, each represented by a letter. The first letter refers to the cardiac chamber that is paced. The second letter designates which chamber is sensed, and the third letter indicates the pacemaker's response to the sensed event. A VVI pacemaker paces the ventricle when the pacemaker fails to sense an intrinsic ventricular depolarization.

3. Which of the following are mechanisms responsible for a myocardial infarction (MI)? (Select all that apply.) a. Coronary artery thrombosis b. Plaque rupture c. Coronary artery spasm near the ruptured plaque d. Preinfarction angina e. Hyperlipidemia

ANS: A, B, C The three mechanisms that block the coronary artery and are responsible for the acute reduction in oxygen delivery to the myocardium are (1) plaque rupture, (2) new coronary artery thrombosis, and (3) coronary artery spasm close to the ruptured plaque.

17. A patient is admitted to the critical care unit with a diagnosis of acute myocardial infarction. The monitor pattern reveals bradycardia. The patient weight is 70 kg and height is 5 feet, 5 inches tall. Based on the above information; occlusion of which coronary artery most likely resulted in bradycardia from sinoatrial node ischemia? a. Right c. Circumflex b. Left anterior descending d. Dominant

ANS: A The right coronary artery provides the blood supply to the sinoatrial and AV nodes in more than half the population. The left coronary artery is a short but important artery that divides into two large arteries, the left anterior descending and the circumflex arteries. These vessels serve the left atrium and most of the left ventricle. The term dominant coronary artery is used to describe the artery that supplies the posterior part of the heart.

34. Which lead is best to monitor a patient? a. Varies based on the patient's clinical condition and recent clinical history b. Lead MCL1 c. Lead V1 d. Lead II

ANS: A The selection of an electrocardiographic monitoring lead is not a decision to be made casually or according to habit. The monitoring lead should be chosen with consideration of the patient's clinical condition and recent clinical history. If the monitored heart has a normal electrical axis, lead II displays a waveform that is predominantly upright, with a positive P wave and positive QRS waveform. P waves are usually easy to identify in lead II, and it is recommended for monitoring of atrial dysrhythmias. However, it is difficult to identify right bundle branch block (RBBB) and left bundle branch block (LBBB). Lead V1 is the optimal lead to select if the critical care nurse needs to analyze ventricular ectopy. V1 provides information to facilitate differentiation between RBBB versus LBBB pattern or distinguish between ventricular tachycardia and supraventricular tachycardia with aberrant conduction, determine whether premature ventricular contractions originate in the right or left ventricle, and clarify when ST segment changes are caused by the RBBB and when they are the result of ischemia. Lead V1 is excellent for this purpose. MCL1 is an uncommon lead choice today. It is used only if monitoring with a three-lead system such as on a transport monitor.

61. A patient is in shock. In this patient, a. cuff pressures may be unreliable. b. cuff pressures are more accurate than A-line pressures. c. cuff pressures and A-line pressures should be nearly identical. d. A-lines should not be inserted.

ANS: A The situation is different if the patient has a low cardiac output or is in shock. The concern is that the cuff pressure may be unreliable because of peripheral vasoconstriction, and an arterial line is generally required.

21. A patient is admitted to the critical care unit with right- and left-sided heart failure. The nurse's assessment reveals that the patient has 3+ pitting edema on the sacrum, blood pressure of 176/98 mm Hg, and bilateral crackles in the lungs. The patient is experiencing shortness of breath and chest discomfort. Increased afterload is probably present related to the patient's blood pressure. Which therapeutic measure will most likely decrease afterload in this patient? a. Administration of vasodilators c. Elevation of extremities b. Placement in high Fowler position d. Increasing intravenous fluids

ANS: A Therapeutic management to decrease afterload is aimed at decreasing the work of the heart with the use of vasodilators. Placing the patient in high Fowler position will cause an increase in the workload of the heart. Elevation of the extremities will ease the venous return back to the heart. Increasing IV fluids will cause an increased workload on the heart.

27. Identify complications with the above rhythm strip. a. Undersensing from a pacemaker c. ICD firing caused by VF b. Oversensing from a pacemaker d. Atrial pacing failure to capture

ANS: A Undersensing is the inability of the pacemaker to sense spontaneous myocardial depolarizations. Undersensing results in competition between paced complexes and the heart's intrinsic rhythm. This malfunction is manifested on the electrocardiogram by pacing artifacts that occur after or are unrelated to spontaneous complexes. Oversensing occurs as a result of inappropriate sensing of extraneous electrical signals that leads to unnecessary triggering or inhibition of stimulus output, depending on the pacer mode. The source of these electrical signals can range from tall peaked T waves to external electromagmetic interference in the critical care environment. The implantable cardioverter defibrillator system consists of leads and a generator and is similar to a pacemaker but with some key differences. The leads contain not only electrodes for sensing and pacing but also integrated defibrillator coils capable of delivering a shock. If the pacing stimulus fires but fails to initiate a myocardial depolarization, a pacing artifact will be present but will not be followed by the expected P wave.

69. The patient's admitting 12-lead ECG shows wide, M-shaped P waves. Which of the following admitting diagnoses could be responsible for this finding? a. Mitral stenosis c. Hypotension b. Chronic pulmonary disease d. Pericarditis

ANS: A Wide, M-shaped P waves are seen in left atrial hypertrophy and are called P mitrale because left atrial hypertrophy is often caused by mitral stenosis.

2. Heart murmurs are characterized by which of the following criteria? (Select all that apply.) a. Intensity b. Location c. Quality d. Pitch e. Pathologic cause

ANS: A, B, C, D Murmurs are characterized by specific criteria: Timing is the place in the cardiac cycle (systole/diastole). Location is where it is auscultated on the chest wall (mitral or aortic area). Radiation is how far the sound spreads across chest wall. Quality is whether the murmur is blowing, grating, or harsh. Pitch is whether the tone is high or low. Intensity is the loudness is graded on a scale of 1 through 6; the higher the number, the louder the murmur.

1. Which of the following effects can be associated with physical exercise? (Select all that apply.) a. Decreased LDL cholesterol b. Increased HDL cholesterol c. Decreased triglycerides d. Increased insulin resistance e. Decreased incidence of depression

ANS: A, B, C, E Many research trials have demonstrated the positive effects of physical activity on the other major cardiac risk factors. Exercise alters the lipid profile by decreasing low-density lipoprotein cholesterol and triglyceride levels and increasing high-density lipoprotein cholesterol levels. Exercise reduces insulin resistance at the cellular level, lowering the risk for developing type 2 diabetes, especially if combined with a weight loss program.

1. Which of the following describe(s) S1, the first heart sound? (Select all that apply.) a. It is associated with closure of the mitral and tricuspid valves. b. It is a high-pitched sound. c. It can be heard most clearly with the diaphragm of the stethoscope. d. The best listening point is in the aortic area. e. The "split" sound can best be detected in the tricuspid area.

ANS: A, B, C, E S1 is the sound associated with mitral and tricuspid valve closure and is heard most clearly in the mitral and tricuspid areas. S1 sounds are high pitched and heard best with the diaphragm of the stethoscope.

1. A patient is diagnosed with third-degree heart failure. The nurse reviews the patient's medication list because the following classifications of drugs should be avoided with patients with heart failure. (Select all that apply.) a. NSAIDs b. Antidysrhythmics c. ACE inhibitors d. Calcium channel blockers e. Beta-blockers

ANS: A, B, D Types of medications that have been found to worsen heart failure should be avoided, including most antidysrhythmics, calcium channel blockers, and nonsteroidal anti-inflammatory medications. Angiotensin-converting enzyme inhibitors and beta-blockers are used to treat heart failure.

2. Which of the following clinical manifestations are indicative of left ventricular failure? (Select all that apply.) a. Cool, pale extremities b. Jugular venous distention c. Liver tenderness d. Weak peripheral pulses e. Rales

ANS: A, D, E Patients presenting with left ventricular failure have one of the following: (1) decreased exercise tolerance, (2) fluid retention, or (3) discovery during examination of noncardiac problems. Clinical manifestations of left ventricular failure include decreased peripheral perfusion with weak or diminished pulses; cool, pale extremities; and, in later stages, peripheral cyanosis.

7. The presence of a carotid or femoral bruit may be evidence of a. left-sided heart failure. b. blood flow through a partially occluded vessel. c. the early onset of pulmonary embolism. d. myocardial rupture.

ANS: B A bruit is an extracardiac vascular sound that results from blood flow through a tortuous or partially occluded vessel.

56. A positive signal-averaged ECG indicates that the patient a. has had a heart attack. b. is at increased risk of sudden cardiac death. c. has no heart disease. d. has no need for an electrophysiology study.

ANS: B A positive signal-averaged electrocardiogram (ECG)—in combination with other specific indicators—is a predictor of increased risk for sudden cardiac death. Many patients with a positive signal-averaged ECG (abnormal) display a normal signal-averaged ECG when placed on antidysrhythmic medications. The signal-averaged ECG is not analyzed in isolation. It is used in conjunction with other cardiac diagnostic tests, including the electrophysiology study (EPS). It is a helpful adjunct to the EPS but does not replace it.

28. A patient asks why he had to take a deep breath when the radiology technician took his chest radiograph. Which of the following would be the best response? a. It gets the chest wall closer to the machine. b. Lungs filled with air give a clearer picture. c. It decreases the error caused by motion. d. It makes the heart appear larger.

ANS: B A radiograph is taken when the patient has taken a deep breath (inspiration). During exhalation, the lungs are less full of air, which can make the lung tissue appear "cloudy" as if there is additional lung water. The heart also appears larger during exhalation. This could lead to an erroneous diagnosis of heart failure.

24. A patient with a potassium level of 2.8 mEq/L is given 60 mEq over a 12-hour period, and a repeat potassium level is obtained after the bolus. The current potassium level is 2.9 mEq/L. Which of the following should now be considered? a. Stopping the patient's Aldactone c. Rechecking the potassium level b. Drawing a serum magnesium level d. Monitoring the patient's urinary output

ANS: B A total serum magnesium concentration below 1.5 mEq/L defines hypomagnesemia. It is commonly associated with other electrolyte imbalances, most notably alterations in potassium, calcium, and phosphorus. Low serum magnesium levels can result from many causes.

54. On returning from the cardiac catheterization laboratory, the patient asks if he can get up in the chair. The most appropriate response would be a. "You cannot get up because you may pass out." b. "You cannot get up because you may start bleeding." c. "You cannot get up because you may fall." d. "You cannot get up until you urinate."

ANS: B After catheterization, the patient remains flat for up to 6 hours (varies by institutional protocol and catheter size) to allow the femoral arterial puncture site to form a stable clot. Most bleeding occurs within the first 2 to 3 hours after the procedure.

16. The physiologic effect of left ventricular afterload reduction is a. decreased left atrial tension. c. increased filling pressures. b. decreased systemic vascular resistance. d. decreased cardiac output.

ANS: B Afterload is defined as the pressure the ventricle generates to overcome the resistance to ejection created by the arteries and arterioles. After a decrease in afterload, wall tension is lowered. The technical name for afterload is systemic vascular resistance (SVR). Resistance to ejection from the right side of the heart is estimated by calculating the pulmonary vascular resistance (PVR). The PVR value is normally one-sixth of the SVR.

25. Surgical repair of an AAA occurs when the aneurysm is a. greater than 5 cm and asymptomatic. c. greater than 7 cm and asymptomatic. b. greater than 5 cm and symptomatic. d. greater than 7 cm and symptomatic.

ANS: B An abdominal aortic aneurysm (AAA) is usually repaired when the aneurysm is 5 cm or larger, creating symptoms, or rapidly expanding. This is done to prevent the high mortality rate associated with abdominal rupture.

15. Angiotensin-converting enzyme (ACE) inhibitors such as captopril (Capoten) and enalapril (Vasotec) decrease peripheral vascular resistance and decrease blood pressure by the mechanism of a. direct arterial vasodilation. b. blocking the conversion of angiotensin I to angiotensin II. c. increased fluid excretion at the loop of Henle in the renal tubules. d. peripheral vasoconstriction and central vasodilation.

ANS: B Angiotensin-converting enzyme (ACE) inhibitors produce vasodilation by blocking the conversion of angiotensin I to angiotensin II. Because angiotensin is a potent vasoconstrictor, limiting its production decreases peripheral vascular resistance. In contrast to the direct vasodilators and nifedipine, ACE inhibitors do not cause reflex tachycardia or induce sodium and water retention.

22. During a history examination, a patient tells the nurse, "The cardiologist says I have a leaking valve." The nurse documents that the patient has a history of a. acute mitral regurgitation. c. chronic mitral regurgitation. b. aortic insufficiency. d. pericardial friction rub.

ANS: B Aortic insufficiency is an incompetent aortic valve. If the valve cusps do not maintain this seal, the sound of blood flowing back into the left ventricle during diastole is heard as a decrescendo, high-pitched, blowing murmur. A pericardial friction rub is a sound that can occur within 2 to 7 days after a myocardial infarction. The friction rub results from pericardial inflammation (pericarditis). Classically, a pericardial friction rub is a grating or scratching sound that is both systolic and diastolic, corresponding to cardiac motion within the pericardial sac. Acute mitral regurgitation occurs when the ventricle contracts during systole and a jet of blood is sent in a retrograde manner to the left atrium, causing a sudden increase in left atrial pressure, acute pulmonary edema, and low CO and leading to cardiogenic shock. Chronic mitral regurgitation is auscultated in the mitral area and occurs during systole. It is high pitched and blowing, although the pitch and intensity vary, depending on the degree of regurgitation. As mitral regurgitation progresses, the murmur radiates more widely.

13. Which of the following doses of dopamine (Intropin) results in stimulation of beta1 receptors and increased myocardial contractility? a. 1 mcg/kg/min c. 15 mcg/kg/min b. 5 mcg/kg/min d. 20 mcg/kg/min

ANS: B At low dosages of 1 to 2 mcg/kg/min, dopamine stimulates dopaminergic receptors, causing renal and mesenteric vasodilation. Moderate dosages result in stimulation of beta1 receptors to increase myocardial contractility and improve cardiac output. At dosages greater than 10 mg/kg/min, dopamine predominantly stimulates alpha receptors, resulting in vasoconstriction that often negates both the beta-adrenergic and dopaminergic effects.

7. The contribution of atrial kick to ventricular filling is approximately a. 10%. c. 5%. b. 20%. d. 45%.

ANS: B Atrial contraction, also known as "atrial kick," contributes approximately 20% of blood flow to ventricular filling; the other 80% occurs passively during diastole.

25. Which of the following cardiac enzymes is a highly specific biomarker for myocardial damage? a. CK-MB c. Troponin T b. Troponin I d. LDH

ANS: B Because cTnI is found only in cardiac muscle, it is a highly specific biomarker for myocardial damage, considerably more specific than CK-MB. As a consequence, patients with a positive cTnI result and a negative CK-MB result usually rule in an acute myocardial infarction (MI). A negative cTnI result that remains negative many hours after an episode of chest pain is a strong indicator that the patient is not experiencing an acute MI. Even with a negative cTnI result, symptoms of chest pain still indicate that the patient should have a comprehensive cardiac evaluation to determine if there is underlying CAD present that may later lead to complications.

21. A patient suddenly develops a wide QRS complex tachycardia. The patient's heart rate is 220 beats/min and regular; blood pressure is 96/40 mm Hg; and respiratory rate is 22 breaths/min, and the patient is awake without complaint except for palpitations. Which of the following interventions would be best to try first? a. Adenosine 6 mg rapid IV push c. Verapamil 0.5 mg IV push b. Lidocaine 1 mg/kg IV push d. Digoxin 0.25 mg IV push

ANS: B Because ventricular tachycardia is more dangerous, treatment with lidocaine would be the first intervention. If this rhythm is ventricular tachycardia, lidocaine is the drug of choice. Adenosine and digoxin would have no effect on ventricular tachycardia, and verapamil could cause intractable hypotension. If the rhythm is a supraventricular contraction with aberrant conduction, then lidocaine will do no harm.

21. Which serum lipid value is a significant predictor of future acute MI in persons with established coronary artery atherosclerosis? a. High-density lipoprotein (HDL) c. Triglycerides b. Low-density lipoprotein (LDL) d. Very-low-density lipoprotein

ANS: B Both the LDL-C and total serum cholesterol levels are directly correlated with risk for coronary artery disease, and high levels of each are significant predictors of future acute myocardial infarction in persons with established coronary artery atherosclerosis. LDL-C is the major atherogenic lipoprotein and thus is the primary target for cholesterol-lowering efforts.

19. Which of the following values reflects a normal cardiac output at rest? a. 2.5 L/min c. 7.3 L/min b. 5.8 L/min d. 9.6 L/min

ANS: B Cardiac output is normally expressed in liters per minute (L/min). The normal cardiac output in the human adult is approximately 4 to 8 L/min. It is approximately 4 to 6 L/min at rest and increases with exercise.

31. Mechanical contraction of the heart occurs during which of the following phases of the cardiac cycle? a. Phase 0 c. Phase 3 b. Phase 2 d. Phase 4

ANS: B During phases 1 and 2, an electrical plateau is created, and during this plateau, mechanical contraction occurs. Because there is no significant electrical change, no waveform appears on the electrocardiogram (ECG). During phase 0 (depolarization), the electrical potential changes rapidly from a baseline of ?2-90 mV to +20 mV and stabilizes at about 0 mV. Because this is a significant electrical change, it appears as a wave on the ECG as the QRS. During phase 3 (repolarization), the electrical potential again changes, this time a little more slowly, from 0 mV back to ?2-90 mV. This is another major electrical event and is reflected on the ECG as a T wave. During phase 4 (resting period), the chemical balance is restored by the sodium pump, but because positively charged ions are exchanged on a one-for-one basis, no electrical activity is generated, and no visible change occurs on the ECG tracing.

30. Which anticoagulant(s) enhances activity of antithrombin III and does not require aPTT or ACT monitoring? a. Heparin c. Bivalirudin (Angiomax) b. Enoxaparin (Lovenox) d. Argatroban (Argatroban)

ANS: B Enoxaparin (Lovenox) enhances activity of antithrombin III, a more predictable response than heparin, because enoxaparin is not largely bound to protein. There is no need for activated partial thromboplastin time (aPTT) or ACT monitoring, and there is a lower risk of heparin-induced thrombocytopenia (HIT) than with unfractionated heparin (UFH). Heparin sodium enhances activity of antithrombin III, a natural anticoagulant, to prevent clot formation. The effectiveness of treatment may be monitored by aPTT or ACT. Response is variable because of binding with plasma proteins effects may be reversed with protamine sulfate. Bivalirudin (Angiomax) directly inhibits thrombin. It may be administered alone or in combination with glycoprotein IIb/IIIa inhibitors and produces a dose-dependent increase in aPTT and ACT. It may be used instead of UFH for patients with HIT. Argatroban (Argatroban) directly inhibits thrombin. It may be used instead of UFH for patients with HIT. Whereas ACT is monitored during percutaneous coronary intervention, aPTT is used during prolonged infusion.

14. When assessing the pulmonary arterial waveform, the nurse notices dampening. After tightening the stopcocks and flushing the line, the nurse decides to calibrate the transducer. What are two essential components included in calibration? a. Obtaining a baseline blood pressure and closing the transducer to air b. Leveling the air-fluid interface to the phlebostatic axis and opening the transducer to air c. Having the patient lay flat and closing the transducer to air d. Obtaining blood return on line and closing all stopcocks

ANS: B Ensuring accuracy of waveform calibration of the system includes opening the transducer to air and leveling the air-fluid interface of the transducer to the phlebostatic axis.

2. A patient with heart failure may be at risk for hypomagnesemia as a result of a. pump failure. c. fluid overload. b. diuretic use. d. hemodilution.

ANS: B Hypomagnesemia can be caused by diuresis. Diuretic use with heart failure often contributes to low serum magnesium levels.

6. The hemodynamic effects of a pericardial effusion include a. increased ventricular ejection. c. myocardial ischemia. b. decreased ventricular filling. d. increased afterload.

ANS: B If the fluid collection in the sac (pericardial effusion) impinges on ventricular filling, ventricular ejection, or coronary artery perfusion, a clinical emergency may exist that necessitates removal of the excess pericardial fluid to restore normal cardiac function. Myocardial ischemia is damage of the myocardium muscle as the result of a heart attack.

36. A patient returns from the cardiac catheterization laboratory after angioplasty and stent placement (ECG changes had indicated an inferior wall myocardial infarction in progress). Which lead would best monitor this patient? a. Varies based on the patient's clinical condition and recent clinical history b. Lead MCL1 c. Lead V1 d. Lead II

ANS: B If the monitored heart has a normal electrical axis, lead II displays a waveform that is predominantly upright, with a positive P wave and positive QRS waveform. P waves are usually easy to identify in lead II, and it is recommended for monitoring of atrial dysrhythmias. However, it is difficult to identify right bundle branch block (RBBB) and left bundle branch block (LBBB). The selection of an electrocardiographic monitoring lead is not a decision to be made casually or according to habit. The monitoring lead should be chosen with consideration of the patient's clinical condition and recent clinical history. Lead V1 is the optimal lead to select if the critical care nurse needs to analyze ventricular ectopy. V1 provides information to facilitate differentiation between RBBB versus LBBB pattern or distinguish between ventricular tachycardia and supraventricular tachycardia with aberrant conduction; determine whether premature ventricular contractions originate in the right or left ventricle, and clarify when ST segment changes are caused by the RBBB and when they are the result of ischemia. Lead V1 is excellent for this purpose. MCL1 is an uncommon lead choice today. It is used only if monitoring with a three-lead system such as on a transport monitor.

52. Which of the following AV blocks can be described as a gradually lengthening PR interval until ultimately the final P wave in the group fails to conduct? a. First-degree AV block c. Second-degree AV block, type II b. Second-degree AV block, type I d. Third-degree AV block

ANS: B In Mobitz type I block, the atrioventricular (AV) conduction times progressively lengthen until a P wave is not conducted. This typically occurs in a pattern of grouped beats and is observed on the electrocardiogram (ECG) by a gradually lengthening PR interval until ultimately the final P wave in the group fails to conduct. When all atrial impulses are conducted to the ventricles but the PR interval is greater than 0.20 second, a condition known as first-degree AV block exists. Mobitz type II block is always anatomically located below the AV node in the bundle of His in the bundle branches or even in the Purkinje fibers. This results in an all-or-nothing situation with respect to AV conduction. Sinus P waves are or are not conducted. When conduction does occur, all PR intervals are the same. Because of the anatomic location of the block, on the surface, ECG the PR interval is constant and the QRS complexes are wide. Third-degree, or complete, AV block is a condition in which no atrial impulses can conduct from the atria to the ventricles. This is also described by the term complete heart block.

29. The most common complication of a central venous catheter (CVC) is a. air embolus. c. thrombus formation. b. infection. d. pneumothorax.

ANS: B Infection related to the use of CVCs is a major problem. The incidence of infection strongly correlates with the length of time the CVC has been inserted, with longer insertion times leading to a higher infection rate. The risk of air embolus, although uncommon, is always present for a patient with a central venous line in place. Air can enter during insertion through a disconnected or broken catheter by means of an open stopcock, or air can enter along the path of a removed CVC. Unfortunately, clot formation (thrombus) at the CVC site is common. Thrombus formation is not uniform; it may involve development of a fibrin sleeve around the catheter, or the thrombus may be attached directly to the vessel wall. Pneumothorax has a higher occurrence during placement of a CVC than during removal.

67. After bathing and repositioning a patient, the nurse notices the following readings on the bedside monitor: PAWP, 20 mm Hg and PAP, 28/12 mm Hg. Assessment notes no changes in patient condition. Which of the following statements would be true? a. The patient may be having a silent MI. b. The monitoring system is not functioning properly. c. The patient is at increased risk for congestive heart failure. d. The patient has pericardial tamponade.

ANS: B It is physically impossible for the pulmonary artery wedge pressure to be higher than the pulmonary artery pressure. Because these readings show this situation, the monitoring system needs troubleshooting.

11. Which of the following statements regarding beta-blockers is correct? a. They increase heart rate and are contraindicated in tachydysrhythmias. b. They result in bronchospasm and should not be used in patients with COPD. c. They increase cardiac output and help with left ventricular failure. d. They are helpful in increasing atrioventricular node conduction and are used in heart blocks.

ANS: B Knowledge of the effects of adrenergic-receptor stimulation allows for anticipation of not only the therapeutic responses brought about by beta-blockade but also the potential adverse effects of these agents. For example, bronchospasm can be precipitated by noncardioselective beta-blockers in a patient with chronic obstructive pulmonary disease secondary to blocking the effects of beta2 receptors in the lungs.

23. Which calcium channel blocker is easily titrated and beneficial in treating hypertension in patients with CAD or CVA? a. Nifedipine (Procardia) c. Clevidipine (Cleviprex) b. Nicardipine (Cardene) d. Diltiazem (Cardizem)

ANS: B Nicardipine was the first available intravenous calcium channel blocker and as such could be more easily titrated to control blood pressure. Because this medication has vasodilatory effects on coronary and cerebral vessels, it has proven beneficial in treating hypertension in patients with coronary artery disease or ischemic stroke. Nifedipine is available only in an oral form, but in the past it was prescribed sublingually during hypertensive emergencies. Clevidipine is a new, short acting calcium channel blocker that allows for even more precise titration of blood pressure in the management of acute hypertension. Diltiazem (Cardizem) is from the benzothiazine group of calcium channel blockers. These medications dilate coronary arteries but have little effect on the peripheral vasculature. They are used in the treatment of angina, especially that which has a vasospastic component, and as antidysrhythmics in the treatment of supraventricular tachycardias.

1. A patient with a serum potassium level of 6.8 mEq/L may exhibit electrocardiographic changes of a. a prominent U wave. c. a narrowed QRS. b. tall, peaked T waves. d. sudden ventricular dysrhythmias.

ANS: B Normal serum potassium levels are 3.5 to 4.5 mEq/L. Tall, narrow peaked T waves are usually, although not uniquely, associated with early hyperkalemia and are followed by prolongation of the PR interval, loss of the P wave, widening of the QRS complex, heart block, and asystole. Severely elevated serum potassium (greater than 8 mEq/L) causes a wide QRS tachycardia.

10. When an intra-aortic balloon is in place, it is essential for the nurse to frequently assess a. for a pulse deficit. b. peripheral pulses distal to the catheter insertion site. c. bilateral blood pressures. d. coronary artery perfusion.

ANS: B One complication of intra-aortic balloon support is lower extremity ischemia resulting from occlusion of the femoral artery by the catheter itself or by emboli caused by thrombus formation on the balloon. Although ischemic complications have decreased with sheathless insertion techniques and the introduction of smaller balloon catheters, evaluation of peripheral circulation remains an important nursing assessment. The presence and quality of peripheral pulses distal to the catheter insertion site are assessed frequently along with color, temperature, and capillary refill of the involved extremity. Signs of diminished perfusion must be reported immediately.

3. Which of the following diagnostic tests is most effective for measuring overall heart size? a. Twelve-lead electrocardiography c. Chest radiography b. Echocardiography d. Vectorcardiography

ANS: C Chest radiography is the oldest noninvasive method for visualizing images of the heart, and it remains a frequently used and valuable diagnostic tool. Information about cardiac anatomy and physiology can be obtained with ease and safety at a relatively low cost. Radiographs of the chest are used to estimate the cardiothoracic ratio and measure overall heart size.

4. A patient reports feeling dizzy after standing quickly. Which of the following could provide a clue regarding the cause? a. Hemoglobin level of 14.0 g/dL and hematocrit level of 42.3% b. Poor skin turgor with extended tenting c. Supine blood pressure of 146/93 mm Hg d. Resting heart rate of 96 beats/min

ANS: B Poor skin turgor could suggest dehydration. Dehydration can cause orthostatic hypotension because of low capacitance reserves from hypovolemia. Supine blood pressure of 146/93 mm Hg would be considered hypertensive, and the patient would most likely experience a headache rather than dizziness. A resting heart rate of 96 beats/min is still considered a normal value.

10. An essential aspect of teaching that may prevent recurrence of heart failure is a. notifying the physician if a 2-lb weight gain occurs in 24 hours. b. compliance with diuretic therapy. c. taking nitroglycerin if chest pain occurs. d. assessment of an apical pulse.

ANS: B Primary topics of education include (1) the importance of a daily weight, (2) fluid restrictions, and (3) written information about the multiple medications used to control the symptoms of heart failure. Reduction or cessation of diuretics usually results in sodium and water retention, which may precipitate heart failure.

16. The nurse assesses the dorsalis pedis and posterior tibial pulses as weak and thready. Indicate the correct documentation for the pulse volume that the nurse would use. a. 0 c. 2+ b. 1+ d. 3+

ANS: B Pulse volumes are 0, not palpable; 1+, faintly palpable (weak and thready); 2+, palpable (normal pulse); and 3+, bounding (hyperdynamic pulse).

15. A heart murmur is described as blowing, grating, or harsh. This description would fall under which criteria? a. Intensity c. Timing b. Quality d. Pitch

ANS: B Quality is whether the murmur is blowing, grating, or harsh. Intensity is the loudness graded on a scale of 1 through 6; the higher the number, the louder is the murmur. Timing is the place in the cardiac cycle (systole/diastole). Pitch is whether the tone is high or low.

41. What is the initial intervention in a patient with sinus tachycardia with the following vital signs: HR, 136 beats/min; BP, 102/60 mm Hg; RR, 24 breaths/min; T, 99.2° F; SpO2, 94% on oxygen 2 L/min by nasal cannula? a. Stat adenosine to decrease heart rate b. Identification and correction of the cause of the increased heart rate c. Sublingual nitroglycerine 0.4 mg d. Lidocaine 75 mg IV push

ANS: B Sinus tachycardia can be caused by a wide variety of factors, such as exercise, emotion, pain, fever, hemorrhage, shock, heart failure, and thyrotoxicosis. Illegal stimulant drugs such as cocaine, "ecstasy," and amphetamines can raise the resting heart rate significantly. Many medications used in critical care can also cause sinus tachycardia; common culprits are aminophylline, dopamine, hydralazine, atropine, and catecholamines such as epinephrine. This patient has a stable heart rate and SpO2; therefore, there is time to identify the cause of the sinus tachycardia. Lidocaine is indicated for ventricular dysrhythmias. Nitroglycerine is not indicated because the patient is not having chest pain at this time. Adenosine is usually not indicated unless the heart rate is greater than 150 beats/min.

28. The predominant form of percutaneous coronary intervention (PCI) is a. Rotablator. b. stents. c. directional coronary atherectomy (DCA). d. balloon angioplasty (PTCA).

ANS: B Stents are currently the predominant form of percutaneous coronary intervention and are used in more than 90% of all interventional procedures. The Rotablator device has a high-speed, rotating, diamond-coated bur that drills through the plaque, creating tiny particles. Because directional coronary atherectomy extracts pieces of atheroma that can be studied microscopically (similar to a biopsy specimen), it has contributed significantly to our understanding of atherosclerosis and restenosis. Percutaneous transluminal coronary angioplasty (PTCA), frequently abbreviated to balloon angioplasty or simply angioplasty, was introduced in 1977 as an alternative to coronary surgical revascularization. PTCA avoided many of the risks associated with cardiac surgery (general anesthesia, sternotomy, extracorporeal circulation, and mechanical ventilation), but its success was hampered by complications related to the procedure (acute closure) and restenosis or renarrowing of the vessel after the procedure.

48. Ventricular tachycardia has which of the following hemodynamic effects? a. Decreased cardiac output from increased ventricular filling time b. Decreased cardiac output from decreased stroke volume c. Decreased cardiac output from increased preload d. Decreased cardiac output from decreased afterload

ANS: B Tachycardia is detrimental to anyone with ischemic heart disease because it decreases the time for ventricular filling, decreases stroke volume, and compromises cardiac output. Tachycardia increases heart work and myocardial oxygen demand while decreasing oxygen supply by decreasing coronary artery filling time.

37. The patient's admitting 12-lead ECG shows peaked P waves. Which of the following admitting diagnoses could be responsible for this finding? a. Mitral stenosis c. Ischemia b. Pulmonary edema d. Pericarditis

ANS: B Tall, peaked P waves occur in right atrial hypertrophy and are referred to as P pulmonale because this condition is often the result of chronic pulmonary disease. Ischemia occurs when the delivery of oxygen to the tissues is insufficient to meet metabolic demand. Cardiac ischemia in an unstable form occurs because of a sudden decrease in supply, such as when the artery is blocked by a thrombus or when coronary artery spasm occurs. If the pulmonary edema is caused by heart failure, sometimes described as hydrostatic pulmonary edema, the fluid may be in a "bat-wing" distribution, with the white areas concentrated in the hilar region (origin of the major pulmonary vessels). However, as the heart failure progresses, the quantity of fluid in the alveolar spaces increases, and the white, fluffy appearance is seen throughout the lung. Pericarditis is inflammation of the sac around the heart.

2. The atrioventricular (AV) node delays the conduction impulse from the atria (0.8-1.2 seconds) for which of the following reasons? a. To limit the amount of blood that fills the ventricle from the atria b. To provide time for the ventricles to fill during diastole c. To limit the number of signals the ventricles receive in some rhythms d. To allow the atria to rest between signals

ANS: B The AV node delays the conduction impulse from the atria (0.8-1.2 seconds) to provide time for the ventricles to fill during diastole.

32. The P wave represents which of the following? a. Atrial contraction c. Sinus node discharge b. Atrial depolarization d. Ventricular contraction

ANS: B The P wave is an electrical event and represents atrial depolarization. Atrial contraction should accompany the P wave but does not always. The sinus node discharge is too faint to be recorded on the surface electrocardiogram. Ventricular contraction usually accompanies the QRS complex.

10. Which portion of the ECG is most valuable in diagnosing atrioventricular (AV) conduction disturbances? a. P wave c. QRS complex b. PR interval d. QT interval

ANS: B The PR interval is an indicator of atrioventricular nodal function. The P wave represents atrial depolarization. The QRS complex represents ventricular depolarization, corresponding to phase 0 of the ventricular action potential. The QT interval is measured from the beginning of the QRS complex to the end of the T wave and indicates the total time interval from the onset of depolarization to the completion of repolarization.

10. An 82-year-old patient is admitted into the critical care unit with a diagnosis of left-sided heart failure related to mitral stenosis. Physical assessment findings reveal tachycardia with an S3 and a 3/6 systolic murmur. The nurse knows that the presence of an S3 heart sound is a. normal for a person this age. b. a ventricular gallop. c. a systolic sound. d. heard best with the diaphragm of the stethoscope.

ANS: B The abnormal heart sounds are labeled the third heart sound (S3) and the fourth heart sound (S4) and are referred to as gallops when auscultated during an episode of tachycardia. Not unexpectedly, the development of an S3 heart sound is strongly associated with elevated levels of brain natriuretic peptide.

42. A patient presents with atrial flutter with an atrial rate of 280 beats/min and a ventricular rate of 70 beats/min. Which of the following best explains this discrepancy in rates? a. The ventricles are too tired to respond to all the atrial signals. b. The AV node does not conduct all the atrial signals to the ventricles. c. Some of the atrial beats are blocked before reaching the AV node. d. The ventricles are responding to a ventricular ectopic pacemaker.

ANS: B The atrioventricular (AV) node does not allow conduction of all these impulses to the ventricles. In this case, the rhythm would be described as atrial flutter with a 4:1 AV block, indicating that only one of every four atrial signals is conducted to the ventricles.

9. Which of the following mechanisms is responsible for the augmentation of coronary arterial blood flow and increased myocardial oxygen supply seen with the intra-aortic balloon pump? a. The vacuum created in the aorta as a result of balloon deflation b. Diastolic inflation with retrograde perfusion c. Forward flow to the peripheral circulation d. Inflation during systole to augment blood pressure

ANS: B The blood volume in the aorta below the level of the balloon is propelled forward toward the peripheral vascular system, which may enhance renal perfusion. Subsequently, the deflation of the balloon just before the opening of the aortic valve creates a potential space or vacuum in the aorta, toward which blood flows unimpeded during ventricular ejection. This decreased resistance to left ventricular ejection, or decreased afterload, facilitates ventricular emptying and reduces myocardial oxygen demands.

13. Depolarization of one myocardial cell will likely result in a. completion of the action potential in that cell before a new cell can accept an impulse. b. quick depolarization and spread to all of the heart. c. depolarization of only cells superior to the initial depolarization. d. depolarization of only cells inferior to the initial depolarization.

ANS: B The cardiac muscle is a functional syncytium in which depolarization started in any cardiac cell is quickly spread to all of the heart.

16. The classic description of pain associated with aortic dissection is a. substernal pressure. c. numbness and tingling in the left arm. b. tearing in the chest, abdomen, or back. d.

ANS: B The classic clinical presentation is the sudden onset of intense, severe, tearing pain, which may be localized initially in the chest, abdomen, or back. As the aortic tear (dissection) extends, pain radiates to the back or distally toward the lower extremities. Many patients have hypertension upon initial presentation, and the focus is on control of blood pressure and early operation.

9. Which step of impulse conduction is most conducive to atrial kick? a. The firing of the sinoatrial node, which results in atrial depolarization b. The conduction delay at the AV node, allowing time for filling c. Conduction through the bundle of His, enhancing ventricular depolarization d. Conduction to the Purkinje fibers, allowing for ventricular contraction

ANS: B The conduction delay at the AV node allows adequate time for ventricular filling from atrial contraction.

11. The outermost layer of the artery that helps strengthen and shape the vessel is the a. tunica. c. adventitia. b. intima. d. media.

ANS: C The adventitia is the outermost layer of the artery that helps strengthen and shape the vessel. The media is the middle layer that is made up of smooth muscle and elastic tissue. The intima is the innermost layer consists of a thin lining of endothelium and a small amount of elastic tissue.

58. Which of the following expresses the correct order when working with an invasive pressure monitor? a. Level the transducer, locate the phlebostatic axis, zero the transducer, and take the reading. b. Locate the phlebostatic axis, level the transducer, zero the transducer, and take the reading. c. Take the reading, level the transducer, locate the phlebostatic axis, and zero the transducer. d. Locate the phlebostatic axis, zero the transducer, level the transducer, and take the reading.

ANS: B The correct order is locate the phlebostatic axis, level the transducer, zero the transducer, and take the reading. The transducer cannot be zeroed before it is leveled. Readings cannot be taken before the transducer is zeroed, and leveling the transducer cannot occur until the phlebostatic axis has been identified.

6. The major key to the clinical significance of atrial flutter is the a. atrial rate. c. PR interval. b. ventricular response rate. d. QRS duration.

ANS: B The major factor underlying atrial flutter symptoms is the ventricular response rate. If the atrial rate is 300 and the atrioventricular (AV) conduction ratio is 4:1, the ventricular response rate is 75 beats/min and should be well tolerated. If, on the other hand, the atrial rate is 300 beats/min but the AV conduction ratio is 2:1, the corresponding ventricular rate of 150 beats/min may cause angina, acute heart failure, or other signs of cardiac decompensation.

19. In analyzing the ECG strip, the nurse notices a spike before each QRS complex. The patient's heart rate is 70 beats/min. This phenomenon is reflective of a. 60-cycle electrical interference; check equipment. b. pacing artifact; the pacemaker is sensing and capturing. c. electrical artifact; the pacemaker is not sensing. d. patient movement; check electrodes.

ANS: B The pacing artifact is the spike that is seen on the electrocardiographic tracing as the pacing stimulus is delivered to the heart. A P wave is visible after the pacing artifact if the atrium is being paced. Similarly, a QRS complex follows a ventricular pacing artifact. With dual-chamber pacing, a pacing artifact precedes both the P wave and the QRS complex.

5. Which of the following criteria are representative of the patient in normal sinus rhythm? a. Heart rate, 64 beats/min; rhythm regular; PR interval, 0.10 second; QRS, 0.04 second b. Heart rate, 88 beats/min; rhythm regular; PR interval, 0.18 second; QRS, 0.06 second c. Heart rate, 54 beats/min; rhythm regular; PR interval, 0.16 second; QRS, 0.08 second d. Heart rate, 92 beats/min; rhythm irregular; PR interval, 0.16 second; QRS, 0.04 second

ANS: B The parameters for normal sinus rhythm are heart rate, 60 to 100 beats/min; rhythm, regular; PR interval, 0.12 to 0.20 second; and QRS, 0.06 to 0.10 second.

1. The possibility of microshock when handling a temporary pacemaker can be minimized by a. decreasing the milliamperes. b. wearing gloves. c. positioning the patient on the left side. d. wearing rubber-soled shoes.

ANS: B The possibility of "microshock" can be minimized by wearing gloves when handling the pacing wires and by proper insulation of terminal pins of pacing wires when they are not in use. The latter can be accomplished either by using caps provided by the manufacturer or by improvising with a plastic syringe or section of disposable rubber glove. The wires are to be taped securely to the patient's chest to prevent accidental electrode displacement.

19. The value of SVO2 monitoring is to determine a. oxygen saturation at the capillary level. b. an imbalance between oxygen supply and metabolic tissue demand. c. the diffusion of gases at the alveolar capillary membrane. d. the predicted cardiac output for acute pulmonary edema.

ANS: B Three of these factors (CO, Hgb, and Sao2) contribute to the supply of oxygen to the tissues. Tissue metabolism (Vo2) determines oxygen consumption or the quantity of oxygen extracted at tissue level that creates the demand for oxygen.

49. The patient is admitted to the ICU with a diagnosis of cardiogenic shock. The patient's VS includes HR 135 beats/min with weak peripheral pulses. The lungs have crackles in the bases bilaterally. O2 saturation is 90% on 4L/NC. The physician orders diuretics and vasodilators. The nurse expects which responses to the medications. a. ?3? SVR, ?3? preload c. ?3? SVR, ?5? preload b. ?5? SVR, ?5? preload d. ?5? SVR, ?3? preload

ANS: B Vasodilators or intra-aortic balloon pumps are used to decrease systemic vascular resistance and diuretics to decrease preload.

44. Which of the following is most often found in ventricular dysrhythmias? a. Retrograde P waves c. No P waves b. Wide QRS complexes d. An inverted T wave

ANS: B Ventricular dysrhythmias result from an ectopic focus in any portion of the ventricular myocardium. The usual conduction pathway through the ventricles is not used, and the wave of depolarization must spread from cell to cell. As a result, the QRS complex is prolonged and is always greater than 0.12 second. It is the width of the QRS, not the height, that is important in the diagnosis of ventricular ectopy.

7. The most frequent dysrhythmia seen initially with sudden cardiac death is a. premature ventricular contractions. c. ventricular fibrillation. b. ventricular tachycardia. d. asystole.

ANS: B When the onset of symptoms is rapid, the most likely mechanism of death is ventricular tachycardia, which degenerates into ventricular fibrillation.

A reliable indicator of reperfusion is the appearance of various reperfusion dysrhythmias such as premature ventricular contractions, bradycardia, heart block, and ventricular tachycardia. Rapid resolution of the previously elevated ST segment should occur. The serum concentration of creatine kinase rises rapidly and markedly, a phenomenon termed washout. 3. Nursing interventions after angioplasty would include which of the following? (Select all that apply.) a. Elevating the head of the bed to 45 degrees b. Hydration as a renal protection measure c. Assessing pedal pulses on the involved limb every 15 minutes for the first 2 hours after the procedure d. Monitoring the vascular hemostatic device for signs of bleeding e. Educating the patient on the necessity of staying supine for 1 to 2 hours after the procedure

ANS: B, C, D The head of the bed must not be elevated more than 30 degrees, and the patient should be instructed to keep the affected leg straight. Bed rest is 6 to 8 hours in duration unless a vascular hemostatic device is used. The nurse observes the patient for bleeding or swelling at the puncture site and frequently assesses adequacy of circulation to the involved extremity.

3. Which of the following would indicate successful reperfusion after administration of a fibrinolytic agent? (Select all that apply.) a. Gradual decrease in chest pain b. Intermittent, multifocal premature ventricular contractions c. Rapid resolution of ST elevation d. Rapid rise in creatine kinase MB fraction

ANS: B, C, D A reliable indicator of reperfusion is the appearance of various "reperfusion" dysrhythmias such as premature ventricular contractions, bradycardia, heart block, and ventricular tachycardia. Rapid resolution of the previously elevated ST segment should occur. The serum concentration of creatine kinase rises rapidly and markedly, a phenomenon termed washout.

4. Nursing interventions after angioplasty would include which of the following? (Select all that apply.) a. Elevating the head of the bed to 45 degrees b. Hydration as a renal protection measure c. Assessing pedal pulses on the involved limb every 15 minutes for the first 2 hours after the procedure d. Monitoring the vascular hemostatic device for signs of bleeding e. Educating the patient on the necessity of staying supine for 1 to 2 hours after the procedure

ANS: B, C, D The head of the bed must not be elevated more than 30 degrees, and the patient should be instructed to keep the affected leg straight. Bed rest is 6 to 8 hours in duration unless a vascular hemostatic device is used. The nurse observes the patient for bleeding or swelling at the puncture site and frequently assesses adequacy of circulation to the involved extremity.

2. Which of the following patients would be a candidate for fibrinolytic therapy? (Select all that apply.) a. The patient's chest pain started 8 hours ago. She has a diagnosis of NSTEMI. b. The patient's chest pain started 3 hours ago, and her ECG shows a new left bundle branch block. c. The patient presents to the emergency department with chest pain of 30 minutes' duration. She has a history of cerebrovascular accident 1 month ago. d. The patient has a history of unstable angina. He has been experiencing chest pain with sudden onset. e. The patient's chest pain started 1 hour ago, and his ECG shows ST elevation.

ANS: B, E Eligibility criteria for administering fibrinolytics include chest pain of less than 12 hours' duration and persistent ST elevation. Exclusion criteria include recent surgery, cerebrovascular accident, and trauma.

33. Why is the measurement of the QT interval important? a. It represents ventricular depolarization. b. It represents ventricular contraction. c. An increasing QT interval increases the risk of torsades de pointes. d. A decreasing QT interval increases the risk of torsades de pointes.

ANS: C A prolonged QT interval is significant because it can predispose the patient to the development of polymorphic ventricular tachycardia, known also as torsades de pointes. A long QT interval can be congenital, as a result of genetic inheritance, or it can be acquired from an electrolyte imbalance or medications.

22. Which of the ECG findings would be positive for an inferior wall MI? a. ST segment depression in leads I, aVL, and V2 to V4 b. Q waves in leads V1 to V2 c. Q waves in leads II, III, and aVF d. T-wave inversion in leads V4 to V6, I, and aVL

ANS: C Abnormal Q waves develop in leads overlying the affected area. An inferior wall infarction is seen with changes in leads II, III, and aVF. Leads I and aVF are selected to detect a sudden change in ventricular axis. If ST segment monitoring is required, the lead is selected according to the area of ischemia. If the ischemic area is not known, leads V3 and III are recommended to detect ST segment ischemia.

66. Increased preload a. always increases cardiac output. c. increases cardiac output in some cases. b. never increases cardiac output. d. has no relation to cardiac output.

ANS: C According to the Frank-Starling law of the heart, if preload increases stroke volume, then cardiac output may increase. If, however, preload causes excessive left ventricular stretch, it can actually decrease cardiac output and may result in congestive heart failure.

26. A patient presents with severe substernal chest pain. The patient exclaims, "This is the most severe pain I have ever felt!" The patient reports that the pain came on suddenly about 2 hours ago and that three sublingual nitroglycerin tablets have not relieved the pain. The 12-lead ECG reveals only the following abnormalities: T-wave inversion in leads I, aVL, V4, and V5; pathologic Q waves in leads II, III, and aVF; ST segment elevation in leads V1, V2, V3, and V4. Which of the following is probably accurate about this patient? a. This patient has an old lateral wall infarction. b. This patient is having an inferior wall infarction. c. This patient is having an acute anterior wall infarction. d. This patient is having a posterior wall infarction.

ANS: C Acute anterior wall infarctions are manifested by electrocardiographic (ECG) changes in leads V2 to V4. Inferior infarctions are manifested by ECG changes in leads II, III, and aVF. Lateral wall infarctions are manifested by ECG changes in leads V5 to V6, I, and aVL. Posterior wall infarctions are manifested by ECG changes in leads V1 to V2.

7. A characteristic event in junctional dysrhythmias is a. irregular rhythm. b. rapid depolarization to the ventricles. c. the spread of the impulse in two directions at once. d. a widened QRS.

ANS: C After an ectopic impulse arises in the junction, it spreads in two directions at once. One wave of depolarization spreads upward into the atria and depolarizes them, causing the recording of a P wave on the electrocardiogram. At the same time, another wave of depolarization spreads downward into the ventricles through the normal conduction pathway, producing a normal QRS complex.

46. After an MI, a patient presents with an increasing frequency of PVCs. The patient's heart rate is 110 beats/min, and ECG indicates a sinus rhythm with up to five unifocal PVCs per minute. Which of the following should be done? The patient is alert and responsive and denies any chest pain or dyspnea. a. Administer lidocaine 100 mg bolus IV push stat. b. Administer Cardizem 20 mg IV push stat. c. Notify the physician and monitor the patient closely. d. Nothing; PVCs are expected in this patient.

ANS: C Although premature ventricular contractions (PVCs) are frequently present after myocardial infarction, they are not always benign. In individuals with underlying heart disease, PVCs or episodes of self-terminating ventricular tachycardia (VT) are potentially malignant. Nonsustained VT is defined as three or more consecutive premature ventricular beats at a rate faster than 110 beats/min lasting less than 30 seconds. The patient does not appear symptomatic from the PVCs at this time; therefore, lidocaine is not indicated. Cardizem is not prescribed for ventricular ectopy.

40. To accurately measure the heart rate of a patient in normal sinus rhythm, which technique would be the most accurate? a. The number of R waves in a 6-second strip b. The number of large boxes in a 6-second strip c. The number of small boxes between QRS complexes divided into 1500 d. The number of large boxes between consecutive R waves divided into 300

ANS: C Calculation of heart rate if the rhythm is regular may be done using the following methods. Method 1: number of RR intervals in 6 seconds multiplied by 10 (e.g., 8 × 10 = 80/min). Method 2: number of large boxes between QRS complexes divided into 300 (e.g., 300 ÷ 4 = 75/min). Method 3: number of small boxes between QRS complexes divided into 1500 (e.g., 1500 ÷ 18 = 84/min).

28. The patient is on day 3 of hospitalization in the ICU. The 45-year-old patient was admitted with a diagnosis of acute MI. The patient complains of fatigue, not sleeping the past two nights, and change in appetite. The nurse is aware that these are symptoms of a. angina. c. depression. b. anxiety. d. endocarditis.

ANS: C Depression is a phenomenon that occurs across a wide spectrum of human experience. Key symptoms of depression mentioned frequently by cardiac patients are fatigue, change in appetite, and sleep disturbance.

12. The rationale for giving the patient additional fluids after a cardiac catheterization is that a. fluids help keep the femoral vein from clotting at the puncture site. b. the patient had a nothing-by-mouth order before the procedure. c. the radiopaque contrast acts as an osmotic diuretic. d. fluids increase cardiac output.

ANS: C Fluid is given for rehydration because the radiopaque contrast acts as an osmotic diuretic. Fluid is also used to prevent contrast-induced nephropathy or damage to the kidney from the contrast dye used to visualize the heart structures.

8. A 68-year-old patient is admitted to the critical care unit with reports of midchest pressure radiating into the jaw and shortness of breath when walking up stairs. The patient is admitted with a diagnosis of "rule out myocardial infarction." The history portion of the assessment should be guided by a. medical history. c. presenting symptoms. b. history of prior surgeries. d. a review of systems.

ANS: C For a patient in acute distress, the history taking is shortened to just a few questions about the patient's chief complaint, precipitating events, and current medications. For a patient who is not in obvious distress, the history focuses on the following four areas: review of the patient's present illness; overview of the patient's general cardiovascular status; review of the patient's general health status, including family history of coronary artery disease (CAD), hypertension, diabetes, peripheral arterial disease, or stroke; and survey of the patient's lifestyle, including risk factors for CAD.

3. A patient has an implantable cardioverter defibrillator (ICD) for chronic ventricular tachydysrhythmias. If the patient's rhythm deteriorates to ventricular fibrillation, a. an external defibrillator will need to be applied. b. start CPR and call a code. c. the ICD will defibrillate at a high energy level. d. the ICD is programmed to cardiovert at a high energy level.

ANS: C If the dysrhythmia deteriorates into ventricular fibrillation, the implantable cardioverter defibrillator is programmed to defibrillate at a higher energy. If the dysrhythmia terminates spontaneously, the device will not discharge.

20. A patient is admitted to the critical care unit with right- and left-sided heart failure. The nurse's assessment reveals that the patient has 3+ pitting edema on the sacrum, blood pressure of 176/98 mm Hg, and bilateral crackles in the lungs. The patient is experiencing shortness of breath and chest discomfort. On the basis of this information, how would the nurse evaluate the patient's preload status? a. The patient is hypovolemic and has too little preload. b. The patient is experiencing CHF and has too little preload. c. The patient is experiencing heart failure and has too much preload. d. The patient is hypertensive and the preload is not a factor.

ANS: C Whereas a patient with hypovolemia has too little preload, a patient with heart failure has too much preload.

47. A patient becomes unresponsive. The patient's heart rate is 32 beats/min, idioventricular rhythm; blood pressure is 60/32 mm Hg; SpO2 is 90%; and respiratory rate is 14 breaths/min. Which of the following interventions would the nurse do first? a. Notify the physician and hang normal saline wide open. b. Notify the physician and obtain the defibrillator. c. Notify the physician and obtain a temporary pacemaker. d. Notify the physician and obtain a 12-lead ECG.

ANS: C If the sinus node and the atrioventricular (AV) junction fail, the ventricles depolarize at their own intrinsic rate of 20 to 40 times per minute. This is called an idioventricular rhythm and is naturally protective mechanism. Rather than trying to abolish the ventricular beats, the aim of treatment is to increase the effective heart rate (HR) and re-establish dominance of a higher pacing site such as the sinus node or the AV junction. Usually, a temporary pacemaker is used to increase the HR until the underlying problems that caused failure of the other pacing sites can be resolved.

27. When comparing a portable chest radiograph taken in an intensive care unit bed with one taken with the patient upright in the radiology department, one significant difference is that the a. portable chest radiograph is usually clearer. b. one in the intensive care unit is considered a posterior view. c. portable chest radiograph magnifies some thoracic structures and decreases the sharpness of the structures. d. departmental chest radiograph enlarges some thoracic structures.

ANS: C In the supine radiograph with the patient lying flat on the bed, the x-ray tube can be only approximately 36 inches from the patient's chest because of ceiling height and x-ray equipment construction. This results in a lower quality film from a diagnostic standpoint because the images of the heart and great vessels are magnified and are not as sharply defined.

31. A patient is admitted to the ICU with diagnosis of fever of unknown origin. The patient is complaining of fatigue, malaise, joint pain, and shivering. VS include: T 103F, HR 90 beats/min, RR 22 breaths/min, BP 132/78, and O2 sat 94% on 2L/NC. Blood cultures are ordered. The patient has a history of MI 3 months ago. The nurse suspects that the patient has developed a. CAD. c. endocarditis. b. heart failure. d. pulmonary embolus.

ANS: C Initial symptoms include fever, sometimes accompanied by rigor (shivering), fatigue, and malaise, with up to 50% of patients complaining of myalgias and joint pain. Blood cultures are drawn during periods of elevated temperature.

25. A patient has been newly diagnosed with stable angina. He tells the nurse he knows a lot about his diagnosis already because his father had the same diagnosis 15 years ago. The nurse asks him to state what he already knows about angina. Which of the following responses by the patient indicates the need for additional education? a. He should stop smoking. b. He can no longer drink colas or coffee. c. He can no longer get a strong back massage. d. He should take stool softeners to prevent straining.

ANS: C Longer term education of the patient and the family can begin. Points to cover include (1) risk factor modification, (2) signs and symptoms of angina, (3) when to call the physician, (4) medications, and (5) dealing with emotions and stress. It is essential to teach avoidance of the Valsalva maneuver, which is defined as forced expiration against a closed glottis. This can be explained to the patient as "bearing down" during defecation or breath holding when repositioning in bed. Relaxation therapy and techniques including back rubs are encouraged when appropriate.

75. What is the noninvasive imaging technique which is useful in diagnosing complications of MI? a. 12-lead ECG c. MRI b. CT d. Echocardiography

ANS: C Magnetic resonance imaging is useful in diagnosing complications of myocardial infarction, such as pericarditis or pericardial effusion, valvular dysfunction, ventricular septal rupture, aneurysm, and intracardiac thrombus. Computed tomography is used to calculate the coronary artery calcium score. Echocardiography uses ultrasound reflected best at interfaces between tissues that have different densities. In the heart, these are the blood, cardiac valves, myocardium, and pericardium. Because all these structures differ in density, their borders can be seen on the echocardiogram. The standard 12-lead electrocardiogram provides a picture of electrical activity in the heart using 10 different electrode positions to create 12 unique views of electrical activity occurring within the heart.

21. The nurse should anticipate which of the following as the most common complication after an MI? a. Pulmonary edema c. Dysrhythmias b. Cardiogenic shock d. Deep vein thrombosis

ANS: C Many patients experience complications occurring either early or late in the postinfarction course. These complications may result from electrical dysfunction or from a cardiac contractility problem. Pumping complications can cause heart failure, pulmonary edema, and cardiogenic shock. The presence of a new murmur in a patient with an acute myocardial infarction warrants special attention because it may indicate rupture of the papillary muscle. The murmur can be indicative of severe damage and impending complications such as heart failure and pulmonary edema.

5. Nursing management of the patient with angina is directed toward a. immediate administration of antiplatelet therapy. b. assessment of history of previous anginal episodes. c. assessment and documentation of chest pain episodes. d. administration of prophylactic lidocaine for ventricular ectopy.

ANS: C Nursing interventions focus on early identification of myocardial ischemia, control of chest pain, recognition of complications, maintenance of a calm environment, and patient and family education. It is important to document the characteristics of the pain and the patient's heart rate and rhythm, blood pressure, respirations, temperature, skin color, peripheral pulses, urine output, mentation, and overall tissue perfusion.

27. Three days after an anterior wall MI, a patient is in the critical care unit. The patient is receiving oxygen at 4 L/min by nasal cannula; nitroglycerin paste, 1-inch q6h; and Lopressor 25 mg PO q12h. The monitor shows that the patient is beginning to have premature ventricular contractions (PVCs). Over the course of the next several hours, the PVCs increase in frequency to more than 15 per minute, with occasional runs of multifocal bigeminal PVCs. The patient's vital signs follow: heart rate, 84 beats/min; sinus rhythm with described PVCs; blood pressure, 124/68 mm Hg; respirations, 20 breaths/min; and SpO2, 92%. Laboratory values are blood pH, 7.44; potassium, 4.4 mEq/L; and magnesium, 1.0 mEq/L. Which of the following, if ordered, would be inappropriate for this patient? a. Lidocaine 100 mg IV push b. Increase oxygen to 6 L/min by nasal cannula c. Potassium chloride 40 mEq in 250 mL 0.9% saline IV piggyback over a 4-hour period d. Magnesium sulfate 2 g IV piggyback over a 2-hour period

ANS: C PVCs are initially controlled by administering oxygen to reduce myocardial hypoxia and by correcting acid-base or electrolyte imbalances. In the setting of an acute myocardial infarction, premature ventricular contractions (PVCs) are pharmacologically treated if they have the following characteristics: frequent (>6/min), closely coupled (R-on-T phenomenon), multiform shapes, and occurrence in bursts of three or more, increasing the risk of sustained ventricular tachycardia. The potassium level is within normal limits and replacement is not warranted. All other interventions listed are appropriate for this patient.

8. Assessment of a patient with pericarditis may reveal which of the following signs and symptoms? a. Ventricular gallop and substernal chest pain b. Narrowed pulse pressure and shortness of breath c. Pericardial friction rub and pain d. Pericardial tamponade and widened pulse pressure

ANS: C Pain is the most common symptom of pericarditis, and a pericardial friction rub is the most common initial sign. A friction rub is best auscultated with a stethoscope at the sternal border and is described as a grating, scraping, or leathery scratching. Pericarditis frequently produces a pericardial effusion.

12. Which of the following assessment findings is most specific for acute onset of pulmonary edema? a. Pulmonary crackles c. Pink, frothy sputum b. Peripheral edema d. Elevated central venous pressure

ANS: C Patients experiencing heart failure and pulmonary edema are extremely breathless and anxious and have a sensation of suffocation. They expectorate pink, frothy sputum and feel as if they are drowning. They may sit bolt upright, gasp for breath, or thrash about. The respiratory rate is elevated, and accessory muscles of ventilation are used, with nasal flaring and bulging neck muscles. Respirations are characterized by loud inspiratory and expiratory gurgling sounds.

5. When checking the patient's back, the nurse pushes her thumb into the patient's sacrum. An indentation remains. The nurse charts that the patient has a. sacral compromise. c. pitting edema. b. delayed skin turgor. d. dehydration.

ANS: C Pitting edema occurs when an impression is left in the tissue when the thumb is removed. The dependent tissues within the legs and sacrum are particularly susceptible. Edema may be dependent, unilateral, or bilateral and pitting or nonpitting.

19. A patient's blood pressure is 90/72 mm Hg. What is the patient's pulse pressure? a. 40 mm Hg c. 18 mm Hg b. 25 mm Hg d. 12 mm Hg

ANS: C Pulse pressure describes the difference between systolic and diastolic values. The normal pulse pressure is 40 mm Hg (i.e., the difference between an SBP of 120 mm Hg and a DBP of 80 mm Hg). A patient with a blood pressure of 90/72 mm Hg has a pulse pressure of 18 mm Hg.

1. Data concerning coronary artery disease (CAD) and specific risk factors have demonstrated a. a low correlation of modifiable risk factors to CAD. b. the onset of CAD in middle age. c. an association between development of specific risk factors and CAD. d. no decisive correlation between risk factors and CAD.

ANS: C Research and epidemiologic data collected during the past 50 years have demonstrated a strong association between specific risk factors and the development of CAD. In general, CAD symptoms are seen in persons age 45 years and older. Primary cardiovascular risk factors are different in men and women, with women having higher rates of diabetes and hypertension compared with men.

9. A 68-year-old patient is admitted to the critical care unit with reports of midchest pressure radiating into the jaw and shortness of breath when walking up stairs. The patient is admitted with a diagnosis of "rule out myocardial infarction." When inspecting the patient, the nurse notes that the patient needs to sit in a high Fowler position to breathe. This may indicate a. pericarditis. c. heart failure. b. anxiety. d. angina.

ANS: C Sitting upright to breathe may be necessary for the patient with acute heart failure, and leaning forward may be the least painful position for a patient with pericarditis.

20. A 52-year-old patient presents to the emergency department with reports of substernal chest pain. A history is taken; serum creatine kinase (CK) and lactate dehydrogenase (LDH) isoenzymes and serum lipid studies are ordered, as is a 12-lead ECG. Which of the following results is most significant in diagnosing an MI during the first 12 hours of chest pain? a. ECG—inverted T waves c. Serum enzymes—elevated CK-MB b. Serum enzymes—elevated LDH4 d. Patient history—substernal chest pain

ANS: C The creatine kinase (CK) muscle/brain (MB) biomarker (CK-MB) is released as a result of myocardial damage, and serum levels rise 4 to 8 hours after myocardial infarction (MI), peak at 15 to 24 hours, and remain elevated for 2 to 3 days. Serial samples are drawn routinely at 6- or 8-hour intervals, and three samples are usually sufficient to support or rule out the diagnosis of MI.

15. The final repolarization phase of the action potential is a. phase 1. c. phase 3. b. phase 2. d. phase 4.

ANS: C The final repolarization phase is phase 3 of the action potential. Phases 1 and 2 (partial repolarization) occur as the AP slope returns toward zero. The plateau that follows is described as phase 2. In phase 4 the AP returns to an RMP of -80 to -90 mV.

64. A 55-year-old patient is scheduled for a stress test. The patient's maximal predicted HR is estimated to be a. 65 beats/min. c. 165 beats/min. b. 155 beats/min. d. 265 beats/min.

ANS: C The maximal predicted heart rate is estimated using the formula: 220 ?2- Patient's age: 220 - 55 = 165.

15. The mean arterial pressure (MAP) is calculated by a. averaging three of the patient's blood pressure readings over a 6-hour period. b. dividing the systolic pressure by the diastolic pressure. c. adding the systolic pressure and two diastolic pressures and then dividing by 3. d. dividing the diastolic pressure by the pulse pressure.

ANS: C The mean arterial pressure is one-third systole and two-thirds diastole.

8. The most common complication of fibrinolytic therapy is a. reperfusion chest pain. c. bleeding. b. lethargy. d. heart blocks.

ANS: C The most common complication related to thrombolysis is bleeding.

30. A patient is admitted for palliative care for end-stage heart failure. The nurse's primary goal is a. to reverse heart failure with the use of diuretics. b. to increase activity tolerance. c. symptom management and relief of pain. d. to increase cardiac output related to alteration of contractility.

ANS: C The primary aim of palliative care is symptom management and the relief of suffering. Fundamental to all symptom management strategies for heart failure is the optimization of medications, according to current guidelines.

Which of the following structures is the primary pacemaker of the heart? a. Ventricular tissue c. Sinoatrial node b. Atrioventricular node d. Purkinje fibers

ANS: C With an intrinsic rate of 60 to 100 beats/min, the sinoatrial node is the primary pacemaker in a healthy heart. The atrioventricular node beats 40 to 60 beats/min. Ventricular tissue must have an electrical impulse to contract. Purkinje fibers beat 15 to 40 beats/min.

18. A pacemaker is connected to an external temporary pulse generator. The sensitivity control a. determines heart rate to muscle activity. b. determines the milliamperes needed to achieve capture. c. regulates the ability of the pacemaker to detect the heart's intrinsic electrical activity. d. allows for selection of intervals between atria and ventricles.

ANS: C The sensitivity control regulates the ability of the pacemaker to detect the heart's intrinsic electrical activity. Sensitivity is measured in millivolts (mV) and determines the size of the intracardiac signal that the generator will recognize.

35. When performing a 12-lead ECG, how many wires are connected to the patient? a. 3 c. 10 b. 5 d. 12

ANS: C The standard 12-lead electrocardiogram provides a picture of electrical activity in the heart using 10 different electrode positions to create 12 unique views of electrical activity occurring within the heart. Fours wires are applied to the extremities to produce leads I, II, III, aVR, aVL, and aVF. Six wires are attached to the V1 to V6 chest lead positions.

18. You are teaching a patient with endocarditis about his diagnosis. Which statement below is correct? a. Endocarditis is a viral infection that is easily treated with antibiotics. b. The risk of this diagnosis is occlusion of the coronary arteries. c. A long course of antibiotics is needed to treat this disorder. d. Complications are rare after antibiotics have been started.

ANS: C Treatment requires prolonged IV therapy with adequate doses of antimicrobial agents tailored to the specific infective endocarditis microbe and patient circumstances. Antibiotic treatment is prolonged, administered in high doses intravenously, and may involve combination therapy. Best outcomes are achieved if therapy is initiated before hemodynamic compromise

24. A patient with diagnosis of CAD with chest pain is admitted into the critical care unit. The patient is suddenly awakened with severe chest pain. Three nitroglycerin sublingual tablets are administered 5 minutes apart without relief. A 12-lead ECG reveals nonspecific ST segment elevation. This patient probably has a. silent ischemia. c. unstable angina. b. stable angina. d. Prinzmetal angina.

ANS: C Unstable angina usually is more intense than stable angina, may awaken the person from sleep, or may necessitate more than nitrates for pain relief. A change in the level or frequency of symptoms requires immediate medical evaluation. Severe angina that persists for more than 5 minutes, worsens in intensity, and is not relieved by one nitroglycerin tablet is a medical emergency. Stable angina is predictable and caused by similar precipitating factors each time; typically, it is exercise induced. Patients become used to the pattern of this type of angina and may describe it as "my usual chest pain." Pain control should be achieved within 5 minutes of rest and by taking sublingual nitroglycerin. Silent ischemia describes a situation in which objective evidence of ischemia is observed on an electrocardiographic monitor but the person does not complain of anginal symptoms. Variant unstable angina, or Prinzmetal angina, is caused by a dynamic obstruction from intense vasoconstriction of a coronary artery. Spasm can occur with or without atherosclerotic lesions. Variant angina commonly occurs when the individual is at rest, and it is often cyclic, occurring at the same time every day.

22. Vasopressors are used cautiously with critical care patients because they can cause a. vasoconstriction of the smooth muscles. c. increased afterload of the heart. b. vasodilation of the smooth muscles. d. decreased preload of the heart.

ANS: C Vasopressors are not widely used in the treatment of critically ill cardiac patients because the dramatic increase in afterload is taxing to a damaged heart. Vasopressin, also known as antidiuretic hormone, has become popular in the critical care setting for its vasoconstrictive effects. At higher doses, vasopressin directly stimulates V1 receptors in vascular smooth muscle, resulting in vasoconstriction of capillaries and small arterioles.

23. A patient was admitted on the night shift with a diagnosis of acute myocardial infarction. Upon auscultation, the nurse hears a harsh, holosystolic murmur along the left sternal border. The nurse notifies the physician immediately because the symptoms indicate the patient has developed a. papillary muscle rupture. c. ventricular septal rupture. b. tricuspid stenosis. d. pericardial friction rub.

ANS: C Ventricular septal rupture is a new opening in the septum between the two ventricles. It creates a harsh, holosystolic murmur that is loudest (by auscultation) along the left sternal border. Papillary muscle rupture is auscultation of a new, high-pitched, holosystolic, blowing murmur at the cardiac apex. Tricuspid stenosis is a quiet murmur that becomes louder with inspiration and is located in the epigastrium area. A pericardial friction rub is a sound that can occur within 2 to 7 days after a myocardial infarction. The friction rub results from pericardial inflammation (pericarditis). Classically, a pericardial friction rub is a grating or scratching sound that is both systolic and diastolic, corresponding to cardiac motion within the pericardial sac.

17. A nurse palpates the descending aorta and feels a strong, bounding pulse. The nurse reports the findings to the physician because the results suggest a. decreased cardiac output. c. an aneurysm. b. increased cardiac output. d. aortic insufficiency.

ANS: C When the patient is in the supine position, the abdominal aortic pulsation is located in the epigastric area and can be felt as a forward movement when firm fingertip pressure is applied above the umbilicus. An abnormally strong or bounding pulse suggests the presence of an aneurysm or an occlusion distal to the examination site. If it is prominent or diffuse, the pulsation may indicate an abdominal aneurysm. A diminished or absent pulse may indicate low CO, arterial stenosis, or occlusion proximal to the site of the examination.

21. A patient was admitted to the ICU 3 days ago with a diagnosis of myocardial infarction. The patient is complaining of increased chest pain when coughing, swallowing, and changing positions. The nurse hears a systolic scratching sound upon auscultation of the apical pulse. The nurse notifies the physician. Based on the symptoms, the physician suspects a(n) a. acute mitral regurgitation. c. chronic mitral regurgitation. b. aortic insufficiency. d. pericardial friction rub.

ANS: D A pericardial friction rub is a sound that can occur within 2 to 7 days after a myocardial infarction. The friction rub results from pericardial inflammation (pericarditis). Classically, a pericardial friction rub is a grating or scratching sound that is both systolic and diastolic, corresponding to cardiac motion within the pericardial sac. Acute mitral regurgitation occurs when the ventricle contracts during systole and a jet of blood is sent in a retrograde manner to the left atrium, causing a sudden increase in left atrial pressure, acute pulmonary edema, and low CO and leading to cardiogenic shock. Chronic mitral regurgitation is auscultated in the mitral area and occurs during systole. It is high pitched and blowing, although the pitch and intensity vary, depending on the degree of regurgitation. As mitral regurgitation progresses, the murmur radiates more widely. Aortic insufficiency is an incompetent aortic valve. If the valve cusps do not maintain this seal, the sound of blood flowing back into the left ventricle during diastole is heard as a decrescendo, high-pitched, blowing murmur.

51. The target INR range is a. 1.0 to 2.0. c. 1.5 to 2.5. b. 1.5 to 3.0. d. 2.0 to 3.0.

ANS: D A target international normalized ratio of 2.5 (range, 2.0-3.0) is desirable.

14. Abnormal heart sounds are labeled S3 and S4 and are referred to as __________ when auscultated during a tachycardic episode. a. Korotkoff sounds c. murmurs b. pulse pressure d. gallops

ANS: D Abnormal heart sounds are known as the third heart sound (S3) and the fourth heart sound (S4); they are referred to as gallops when auscultated during an episode of tachycardia. Murmurs are produced by turbulent flood flow through the chambers of the heart, from forward flow through narrowed or irregular valve openings, or backward regurgitate flow through an incompetent valve. Korotkoff sounds are the sounds created by turbulence of blood flow within a vessel caused by constriction of the blood pressure cuff. Pulse pressure describes the difference between systolic and diastolic values. A normal pulse pressure is 40 mm Hg.

2. Which of the following values, when elevated, places the patient at lowest risk for CAD? a. Very-low-density lipoproteins (VLDLs) b. Triglycerides c. Low-density lipoproteins (LDLs) d. High-density lipoproteins (HDLs)

ANS: D All of the reasons are not completely understood, but one recognized physiologic effect is the ability of HDL to promote the efflux of cholesterol from cells. This process may minimize the accumulation of foam cells in the artery wall and thus decrease the risk of developing atherosclerosis. High HDL levels confer both anti-inflammatory and antioxidant benefits on the arterial wall. In contrast, a low HDL level is an independent risk factor for the development of CAD an d other atherosclerotic conditions.

32. At what size is an aortic aneurysm evaluated for surgical repair or stent placement? a. 2 cm c. 5 cm b. 4 cm d. >5 cm

ANS: D An aneurysm smaller than 4 cm in diameter can be managed on an outpatient basis with frequent blood pressure monitoring and ultrasound testing to document any changes in the size of the aneurysm. Management includes weight loss, smoking cessation, and control of hypertension as appropriate. An aortic aneurysm larger than 5 cm in diameter requires evaluation for surgical repair or placement of an aortic stent to eliminate the risk of rupture.

39. A new-onset MI can be recognized by which of the following ECG changes? a. Q waves c. Widened QRS b. Smaller R waves d. ST segment elevation

ANS: D Any change from baseline is expressed in millimeters and may indicate myocardial ischemia (one small box equals 1 mm). ST segment elevation of 1 to 2 mm is associated with acute myocardial injury, preinfarction, and pericarditis. ST segment depression (decrease from baseline more of 1 to 2 mm) is associated with myocardial ischemia. Widened QRS complexes are indicative of ventricular depolarization abnormalities such as bundle branch blocks and ventricular dysrhythmias. Q waves and smaller R waves are indications usually present 24 hours to 1 week after the myocardial infarction is completely evolved; they represent necrosis.

4. Evaluation of arterial circulation to an extremity is accomplished by assessing which of the following? a. Homans sign c. Peripheral edema b. Skin turgor d. Capillary refill

ANS: D Capillary refill assessment is a maneuver that uses the patient's nail beds to evaluate both arterial circulation to the extremity and overall perfusion. The severity of arterial insufficiency is directly proportional to the amount of time necessary to re-establish flow and color.

4. ST segment monitoring for ischemia has gained increasing importance with the advent of thrombolytic therapy. The most accurate method for monitoring the existence of true ischemic changes is a. T-wave inversion in leads overlying the ischemia. b. ST segment depression in leads overlying the ischemia. c. adjusting the gain control on bedside monitoring for best visualization. d. 12-lead ECG for confirmation.

ANS: D Cardiac biomarkers are proteins that are released from damaged myocardial cells. The initial elevation of cTnI, cTnT, and CK-MB occurs 3 to 6 hours after the acute myocardial damage. This means that if an individual comes to the emergency department as soon as chest pain is experienced, the biomarkers will not have risen. For this reason, it is clinical practice to diagnose an acute myocardial infarction by 12-lead electrocardiography and clinical symptoms without waiting for elevation of cardiac biomarkers.

45. The patient has an HR of 84 beats/min and an SV of 65 mL. Calculate the CO. a. 149 mL c. 4650 mL b. 500 mL d. 5460 mL

ANS: D Cardiac output (CO) is the product of heart rate (HR) multiplied by stroke volume (SV). SV is the volume of blood ejected by the heart during each beat (reported in milliliters). 84 x 65 = 5460 mL

17. Contractility of the left side of the heart is measured by a. pulmonary artery wedge pressure. c. systemic vascular resistance. b. left atrial pressure. d. left ventricular stroke work index.

ANS: D Contractility of the left side of the heart is measured by the left ventricular stroke work index.

29. Dyspnea with wheezing, a nonproductive cough, and pulmonary crackles that progress to the gurgling sounds of pulmonary edema is described as a. dyspnea. c. paroxysmal nocturnal dyspnea. b. orthopnea. d. cardiac asthma.

ANS: D Dyspnea with wheezing, a nonproductive cough, and pulmonary crackles that progress to the gurgling sounds of pulmonary edema are symptoms of cardiac asthma. With dyspnea, the patient feels shortness of breath from pulmonary vascular congestion and decreased lung compliance. In orthopnea, the patient has difficulty breathing when lying flat because of an increase in venous return that occurs in the supine position. Paroxysmal nocturnal dyspnea is a severe form of orthopnea in which the patient awakens from sleep gasping for air.

31. A patient is admitted to the ICU after a positive exercise treadmill test with a diagnosis of CAD and stable angina. Radiographic test show that the patient has blockage in the left main coronary artery and four other vessels. The nurse anticipates that the patient will be scheduled for a. medical therapy antianginal medications. c. TAVR. b. PCI. d. CABG.

ANS: D Early studies demonstrated coronary artery bypass graft surgery was more effective than medical therapy for improving survival in patients with left main or three-vessel coronary artery disease and at relieving anginal symptoms. Medical therapy is recommended if the ischemia is prevented by antianginal medications that are well tolerated by the patient. Surgical revascularization has been shown to be more efficacious than percutaneous coronary intervention in patients with multivessel or left main coronary disease. Transcatheter aortic valve replacement is a transformational therapy for patients who have severe aortic stenosis but who are extremely high-risk surgical candidates or who are inoperable by virtue of associated co-morbidities.

14. Patient teaching regarding valvular heart disease should include which of the following? a. Increase fluid intake to increase cardiac output. b. Monitor and increase sodium intake to replace sodium lost with diuretics. c. Increase daily activity until shortness of breath occurs. d. Take prophylactic antibiotics before undergoing any invasive procedure.

ANS: D Education for the patient with acute or chronic heart failure secondary to valvular dysfunction includes (1) information related to diet, (2) fluid restrictions, (3) the actions and side effects of heart failure medications, (4) the need for prophylactic antibiotics before undergoing any invasive procedures such as dental work, and (5) when to call the health care provider to report a negative change in cardiac symptoms.

68. Which of the following conditions can cause an artificial increase in the PAOP/PCWP? a. Aortic regurgitation c. Mitral stenosis b. Aortic stenosis d. Mitral regurgitation

ANS: D If mitral regurgitation is present, the mean pulmonary artery occlusion pressure reading is artificially elevated because of abnormal backflow of blood from the left ventricle to the left atrium during systole.

59. A patient's CVP reading suddenly increased from 10 to 48 mm Hg. His lungs are clear except for fine rales at the bases. Immediate response should be which of the following? a. Nothing; this reading is still within normal limits. b. Place a stat call into the physician. c. Administer ordered prn Lasix. d. Check the transducer level.

ANS: D If the transducer falls below the correct level, the reading would be falsely elevated. This rise is consistent with a transducer having fallen from the correct level on the bed to the floor. Lasix is not indicated. Central venous pressure (CVP) of 45 mm Hg, if true, is severely elevated. Not enough information has been provided to call the physician. If the CVP value is true and the patient's condition is poor, a call to the physician would be appropriate after assessment.

9. A patient with ventricular fibrillation (VF) is a. hypertensive. c. diaphoretic. b. bradypneic. d. pulseless.

ANS: D In VF, the patient does not have a pulse, no blood is being pumped forward, and defibrillation is the only definitive therapy. No forward flow of blood or palpable pulse is present in VF.

14. In a myocardial cell, the normal resting membrane potential is a. 10 to 20 mV. c. -20 to -30 mV. b. 30 to 40 mV. d. -80 to -90 mV.

ANS: D In a myocardial cell, the normal resting membrane potential is -80 to -90 mV.

23. Which of the following cholesterol values indicates a heightened risk for the development of CAD? a. Total cholesterol level of 170 mg/dL c. Triglyceride level of 120 mg/dL b. HDL cholesterol level of 30 mg/dL d. LDL cholesterol level >190 mg/dL

ANS: D Low-density lipoprotein (LDL) cholesterol is usually described as the "bad cholesterol" because high levels are associated with an increased risk of acute coronary syndrome (ACS), stroke, and peripheral arterial disease (PAD). High LDL levels initiate the atherosclerotic process by infiltrating the vessel wall and binding to the matrix of cells beneath the endothelium. Total cholesterol levels below 200 are considered normal. High-density lipoprotein (HDL) cholesterol levels below 40 are at low risk of coronary artery disease. Triglyceride levels below 150 are considered normal.

2. An 82-year-old patient is admitted into the critical care unit with a diagnosis of left-sided heart failure related to mitral stenosis. Physical assessment findings reveal tachycardia with an S3 and a 3/6 systolic murmur. Which of the following descriptions best describes the murmur heard with mitral stenosis? a. High-pitched systolic sound c. High-pitched diastolic sound b. Medium-pitched systolic sound d. Low-pitched diastolic sound

ANS: D Mitral stenosis describes a narrowing of the mitral valve orifice. This produces a low-pitched murmur, which varies in intensity and harshness depending on the degree of valvular stenosis. It occurs during diastole, is auscultated at the mitral area (fifth ICS, midclavicular line), and does not radiate.

38. A nurse is obtaining the history of a patient who reveals that he had an MI 5 years ago. When the admission 12-lead ECG is reviewed, Q waves are noted in leads V3 and V4 only. Which of the following conclusions is most consistent with this situation? a. The patient may have had a posterior wall MI. b. The patient must have had a right ventricular MI. c. The admission 12-lead ECG was done incorrectly. d. The patient may have had an anterior MI.

ANS: D Not every acute myocardial infarction (MI) results in a pathologic Q wave on the 12-lead electrocardiogram (ECG). When the typical ECG changes are not present, the diagnosis depends on symptomatic clinical presentation, specific cardiac biomarkers (e.g., cTnI, cTnT, CK-MB), and non-ECG diagnostic tests such as cardiac catheterization. Anterior and posterior wall MIs have ST changes, not Q wave changes.

18. A nurse from the ICU receives a report from the ED nurse on a patient that includes a diagnosis of syncope unknown etiology. Orthostatic VS lying: 110/80 mm/Hg; sitting: 100/74 mm/Hg; standing: 92/40 mm/Hg. Based on this information, the nurse should monitor the patient's a. breathing. c. peripheral pulses. b. dietary intake. d. activity.

ANS: D Postural (orthostatic) hypotension occurs when the systolic blood pressure drops by 10 to 20 mm Hg or the diastolic blood pressure drops by 5 mm Hg after a change from the supine posture to the upright posture. This is usually accompanied by dizziness, lightheadedness, or syncope. If a patient experiences these symptoms, it is important to complete a full set of postural vital signs before increasing the patient's activity level.

11. Which of the following findings would be indicative reasons to abort an exercise stress test? a. Ventricular axis of +90 c. Inverted U wave b. Increase in blood pressure d. ST segment depression or elevation

ANS: D Signs that can alert the nurse to stop the test include ST segment elevation equal to or greater than 1.0 mm (one small box) or ST depression equal to or greater than 2.0 mm (2 small boxes). Blood pressure is expected to rise during exercise, but a systolic blood pressure greater than 250 mm Hg or a diastolic blood pressure greater than 115 mm Hg is considered high enough to stop the test. Parameters for ventricular axis in degrees are ?2-30° to +90°. Left-axis deviation is present if the axis falls between ?2-30° and ?2-90°.

22. Which classification of dysrhythmia is most common with an inferior wall infarction in the first hour after STEMI? a. Sinus tachycardia c. Atrial fibrillation b. Multifocal PVCs d. Sinus bradycardia

ANS: D Sinus bradycardia (heart rate less than 60 beats/min) occurs in 30% to 40% of patients who sustain an acute myocardial infarction (MI). It is more prevalent with an inferior wall infarction in the first hour after ST segment elevation MI. Sinus tachycardia (heart rate more than 100 beats/min) most often occurs with an anterior wall MI. Premature atrial contractions (PACs) occur frequently in patients who sustain an acute MI. Atrial fibrillation is also common and may occur spontaneously or may be preceded by PACs. Premature ventricular contractions (PVCs) are seen in almost all patients within the first few hours after an MI

14. A patient is admitted with left-sided heart failure and a blood pressure of 220/118 mm Hg. Which of the following drugs will be most effective in decreasing the blood pressure and reducing afterload? a. Dopamine (Intropin) c. Propranolol (Inderal) b. Verapamil (Calan) d. Sodium nitroprusside (Nipride)

ANS: D Sodium nitroprusside (Nipride) is a potent, rapidly acting venous and arterial vasodilator, particularly suitable for rapid reduction of blood pressure in hypertensive emergencies and perioperatively. It also is effective for afterload reduction in the setting of severe heart failure. The drug is administered by continuous intravenous infusion, with the dosage titrated to maintain the desired blood pressure and systemic vascular resistance.

15. Which medication may be administered to the patient with hypertensive crisis? a. Digitalis c. Verapamil b. Vasopressin d. Sodium nitroprusside

ANS: D Sodium nitroprusside is frequently the first drug used to lower blood pressure in hypertensive emergency. Sodium nitroprusside is useful because of its half-life of seconds. It is not suitable for long-term use because of development of a metabolite that causes cyanide-like toxicity. Short-acting beta-blockers that are effective are labetalol and esmolol. Beta-blockers are especially effective if aortic dissection is present. Digoxin is frequently prescribed for atrial fibrillation.

3. A patient is admitted to the critical care unit with VS: BP 220/110, P 108, RR 24, T 103F, and O2 sat 94% on oxygen 2L/NC. The patient is responsive and denies chest pain. The physician has ordered a workup for CAD. The nurse expects the admitting diagnosis to be which of the following? a. Hypotension c. Stage 1 hypertension b. Prehypertension d. Stage 2 hypertension

ANS: D Stage 2 hypertension is defined as a systolic blood pressure of 160 mm Hg or above and a diastolic blood pressure of 100 mm Hg or above.

5. The rationale for administration of a fibrinolytic agent is a. dilation of the blocked coronary artery. b. anticoagulation to prevent formation of new emboli. c. dissolution of atherosclerotic plaque at the site of blockage. d. restoration of blood flow to the obstructed coronary artery via lysis of the thrombus.

ANS: D The administration of a fibrinolytic agent results in the lysis of the acute thrombus, thus recanalizing, or opening, the obstructed coronary artery and restoring blood flow to the affected tissue. After perfusion is restored, adjunctive measures are taken to prevent further clot formation and reocclusion.

19. A 66-year-old patient is admitted to the critical care unit with a diagnosis of acute inferior MI. A 12-lead electrocardiogram (ECG) is done to validate the area of infarction. Which finding on the ECG is most conclusive for infarction? a. Inverted T waves c. ST segment depression b. Tall, peaked T waves d. Pathologic Q waves

ANS: D The changes in repolarization are seen by the presence of new Q waves. These new, pathologic Q waves are deeper and wider than tiny Q waves found on the normal 12-lead ECG

3. Why do many patients with very high heart rates frequently have chest pain and shortness of breath? a. Patients with heart disease frequently have an anxiety disorder as well. b. The rapid pounding of the heart in the chest wall causes the physical pain. c. The heart muscle gets tired from the increased work. d. The decreased diastolic time decreases oxygen delivery to the myocardium.

ANS: D The coronary arteries are perfused during diastole. When the heart rate increases, the diastolic time decreases as each contraction has less time to be completed. This decreases the time the coronary arteries have to deliver oxygenated blood to the myocardium. The symptoms described are caused by a lack of oxygen in the myocardium.

12. Which of the following is most descriptive of the capillary? a. Large diameter, low pressure c. Large diameter, high pressure b. Small diameter, high pressure d. Small diameter, low pressure

ANS: D The diameter of a capillary is less than that of an arteriole, but the pressure is relatively low as a result of the large cross-sectional area of the branching capillary bed.

53. The blood test used to standardize PT results among clinical laboratories worldwide is known as a. aPTT. c. HDL. b. ACT. d. INR.

ANS: D The international normalized ratio was developed by the World Health Organization in 1982 to standardize prothrombin time results among clinical laboratories worldwide. High-density lipoproteins are particles of the total serum cholesterol. Activated coagulation time (ACT) is also known as the activated clotting time. The ACT is a point of care test that is performed outside of the laboratory setting in areas such as the cardiac catheterization laboratory, the operating room, or critical care units. The activated partial thromboplastin time is used to measure the effectiveness of intravenous or subcutaneous ultrafractionated heparin therapy.

16. Which statement regarding the autonomic nervous system's role in the regulation of heart rate is true? a. Parasympathetic influences increase heart rate. b. Sympathetic influences are predominantly present. c. Parasympathetic influences are only compensatory. d. Both sympathetic and parasympathetic influences are normally active.

ANS: D The parasympathetic nervous system and the sympathetic nervous system operate to create a balance between relaxation and fight-or-flight readiness. They affect cardiovascular function by slowing the heart rate during periods of calm and increasing it in response to sympathetic stimulation.

24. A patient returns to the ICU after a femorotibial bypass graft of the left leg. Identify the primary focus of nursing care immediately in the postprocedural assessment period. a. Prevention of skin breakdown b. Hourly intake and urinary output c. Monitor for ST segment changes d. Frequent pulse checks to the affected limb

ANS: D The primary focus of nursing care in the immediate postprocedural period is assessment of the adequacy of perfusion to the affected limb and identification of complications. Pulse checks are performed frequently, and the physician is notified of any decrease in the strength of the Doppler signal. Because distal perfusion is compromised in this patient population, nursing measures to prevent skin breakdown are implemented. If the repair was performed above the renal arteries, kidney function may be impaired as a result of interruption of renal blood flow during the procedure. Urine output is therefore assessed hourly and supported with fluids and diuretics as needed. Because patients with peripheral vascular disease are at high risk for cardiac events, ST segment monitoring is performed to detect episodes of myocardial ischemia throughout the perioperative period.

10. Which of the following is an example of a physiologic shunt? a. A ventricular septal defect b. Blood returning from the inferior vena cava to the right atrium c. A septal infarct d. Thebesian vessels returning deoxygenated blood to the left ventricle

ANS: D The thebesian vessels return blood to the left ventricle. The mixing of unoxygenated blood with freshly oxygenated blood is called a physiologic shunt. A ventricular septal defect (VSD) allows mixing of blood from both ventricles. The clinical impact depends on the size of the intracardiac shunt. A VSD is a congenital opening between the ventricles; a ventricular septal rupture can occur as a complication of a large anterior wall myocardial infarction.

18. Which of the following interventions should be strictly followed to ensure accurate cardiac output readings? a. Use 5 mL of iced injectate only. b. Inject the fluid into the pulmonary artery port only. c. Ensure a difference of at least 5° C between injectate temperature and the patient's body temperature. d. Administer the injectate within 4 seconds.

ANS: D To ensure accurate readings, the difference between injectate temperature and body temperature must be at least 10° C, and the injectate must be delivered within 4 seconds, with minimal handling of the syringe to prevent warming of the solution. This is particularly important when iced injectate is used.

16. Noninvasive emergency pacing is best achieved via the use of which type of temporary pacing? a. Transvenous (endocardial) c. Transthoracic b. Epicardial d. Transcutaneous

ANS: D Transcutaneous cardiac pacing involves the use of two large skin electrodes, one placed anteriorly and the other posteriorly on the chest, connected to an external pulse generator. It is a rapid, noninvasive procedure that nurses can perform in the emergency setting and is recommended for the treatment of symptomatic bradycardia.

A nurse is caring for a patient with an anterior wall myocardial infarction. The patient continues to have chest pain and signs of impending heart failure. The nurse consults with the practitioner regarding placement of the patient on an intraaortic balloon pump (IABP). What is the goal of this therapy? Reduce preload and increase afterload. Reduce afterload and increase coronary blood flow. Dilate coronary arteries and improve contractility. Improve contractility and reduce afterload.

B Intraaortic balloon pump (IABP) improves perfusion to coronary arteries by inflation of the balloon during diastole, when coronary arteries receive their blood flow. By deflating the balloon just before systole, the IABP decreases the resistance, or afterload, against which the ventricle ejects its blood. Reducing preload helps with heart failure, but increasing afterload requires the weakened heart to work harder, which worsens heart failure. The IABP does not dilate coronary arteries or affect contractility. Reducing afterload improves cardiac output in a heart that is failing, but the IABP does not affect contractility. (EVOLVE)

After a coronary artery bypass graft (CABG), the nurse notes the patient has muffled heart tones, jugular vein distention, and decreased blood pressure. There is no drainage from the chest tube for the last 2 hours. The nurse suspects the patient has developed which complication? Coronary vasospasm Cardiac tamponade Postoperative infection Postoperative hemorrhage

B These are the signs of cardiac tamponade. Symptoms of coronary vasospasm include chest pain. Symptoms of postoperative infection include fever, drainage, and tachycardia. Symptoms of postoperative hemorrhage include flattened neck veins, tachycardia, and increased chest tube drainage.Cognitive Level: EvaluatingNursing Process: AssessmentClient Need: Physiological Integrity: Physiological Adaptation (EVOLVE)

A patient is admitted with an acute myocardial infarction. Upon assessment, the patient is noted to be confused. The nurse suspects that the confusion is most likely attributable to which cause? A Early onset of dementia B Low cardiac output C Anxiety over chest pain D Poor oxygen exchange

B (EVOLVE)

A patient recovering from a myocardial infarction (MI) notifies the nurse that he is having chest pain. Upon listening to the patient's heart sounds, the nurse hears a grating sound that is present during both systole and diastole. The nurse suspects that the patient's chest pain is most likely caused by which condition? A Another MI B Inflammation of the pericardium C Papillary muscle rupture D Ventricular septal rupture

B (EVOLVE)

A patient who is in the cardiac intensive care unit with an acute myocardial infarction (AMI) suddenly becomes hypotensive, tachycardic, and short of breath. Upon further assessment, the nurse hears a high-pitched holosystolic blowing murmur. The nurse is concerned that the patient may have developed which complication? A Cardiac tamponade B Papillary muscle rupture C Pericardial friction rub D Ventricular septal rupture

B (EVOLVE)

Aortic valve dysfunction pathologically alters the which structure of the heart? A Left atrium B Left ventricle C Right ventricle D Mitral valve

B (EVOLVE)

Arterial blood pressure monitoring is used for the assessment of cardiac output, fluid status, and tissue perfusion. Which parameter is observed with arterial blood pressure monitoring? A End-diastolic pulmonary pressure B Mean arterial pressure C Peripheral vascular resistance D Mixed venous oxygenation

B (EVOLVE) Intraarterial blood pressure monitoring is indicated for any major medical or surgical condition that compromises cardiac output, tissue perfusion, or fluid volume status. The system is designed for continuous measurement of three blood pressure parameters: systole, diastole, and mean arterial blood pressure. The direct arterial access is helpful in the management of patients with acute respiratory failure who require frequent arterial blood gas measurements.

A new graduate asks the preceptor why the hospital is not using heparin in the hemodynamic flush bags. What is the correct response? "It is a cost-saving measure." "There is a correlation between heparin infusions in cardiac patients and heparin-induced thrombocytopenia." "Heparin infusions increase the occurrence of clots, so we no longer use them." "The pressure on the bag breaks down the heparin, so heparin infusions are not effective."

B (EVOLVE) A patient receiving heparin is at increased risk for heparin-induced thrombocytopenia. Although it may save money, this is not the reason many hospitals have stopped using heparin. Heparin decreases, not increases, the occurrence of clots. Pressure does not destabilize heparin

The nurse is choosing the appropriate lead to monitor the patient's electrocardiogram rhythm. For which patient would lead II be the most appropriate lead? AA patient with a right bundle branch block B A patient in atrial fibrillation C A patient who is having supraventricular versus ventricular tachycardia D A postintervention recovery patient after a stent placement for a lateral infarct

B (EVOLVE) Atrial rhythms are best monitored in lead II. Lead II would not help with identification of a right bundle branch block. MCL1 is the best lead for differentiation of tachycardias. V6 is the best lead for monitoring a patient with a lateral infarct.Cognitive Level: ApplyingNursing Process: AssessmentClient Need: Physiological Integrity: Reduction of Risk Potential

A nurse admits a patient from the emergency department with a diagnosis of acute coronary syndrome (ACS) and anterior wall myocardial infarction (MI). Per physician order, the nurse administers oxygen and nitroglycerin. This therapy should reduce or relieve chest pain by which mechanism? Increasing preload and decreasing afterload Increasing oxygen delivery and decreasing oxygen demand Minimizing plaque formation and preventing vasospasm Preventing dysrhythmias and decreasing cardiac contractility

B In the acute period, if severe heart muscle damage has occurred, myocardial oxygen supply is increased by the administration of supplemental oxygen to prevent tissue hypoxia. Myocardial oxygen supply can be further enhanced by the use of coronary artery vasodilators. Nitroglycerin is recommended for the first 48 hours to increase vasodilatation and prevent myocardial ischemia. (EVOLVE)

A patient woke up from a sound sleep in a cold sweat with nausea and light-headedness and now has chest pain (8 of 10 on the pain scale) that is unrelieved by nitroglycerin (NTG) after 5 minutes. The nurse suspects the patient is experiencing which problem? Stable angina Unstable angina Variant angina Silent ischemia

B The patient is showing signs of unstable angina. Stable angina occurs with predictable precipitating factors and improves with rest or nitroglycerin (NTG) within 5 minutes. Variant angina is caused by spasm of a coronary artery, usually occurs at the same time every day, and is relieved by NTG. Silent ischemia is painless.Cognitive Level: UnderstandingNursing Process: AssessmentClient Need: Physiological Integrity: Physiological Adaptation (EVOLVE)

When caring for a patient postoperatively after aortic aneurysm repair, the nurse must closely monitor the patient for hypertension because it can cause which complication? Ischemic stroke Graft disruption Tachycardia Pulmonary edema

B In the immediate postoperative period, hypertension, which is the most common cause of aortic aneurysm, can cause bleeding at the site of the graft. If hypertension occurs, it may require treatment with a continuous infusion of a vasodilating drug. Ischemic stroke is not a complication of aortic aneurysm repair unless the aneurysm is located close to the aortic arch; even then, it is uncommon. Tachycardia may occur if the patient has inadequate pain control but is not a complication of the surgery itself. Pulmonary edema is a consequence of hypertensive crisis; monitoring and controlling hypertension prevent any chance of the hypertension becoming that severe. (EVOLVE)

The charge nurse has just received a report on the patients in the cardiac intervention recovery unit. Which patient should be seen first? Patient 3 hours after percutaneous coronary angioplasty (PTCA) with an ACT of 180 seconds Patient with an arterial and venous sheath intact via the left leg with no pedal pulse Patient who had sheath removed 6 hours ago and is asking for a urinal Patient who denies chest pain after a PTCA

B The patient with the sheaths in place is experiencing circulatory compromise as evidenced by the loss of pedal pulse in the extremity. This patient would be considered top priority because of the potential loss of blood flow to the patient's leg. The patient with the ACT of 180 seconds will need the sheath removed, but this can wait for a few more minutes. The patient who needs a urinal can also wait a few more minutes. The patient who denies chest pain is stable.Cognitive Level: EvaluatingNursing Process: PlanningClient Need: Physiological Integrity: Reduction of Risk Potential (EVOLVE)

A patient in the acute phase of systolic heart failure is admitted to the intensive care unit. Which interventions would the nurse anticipate? Select all that apply. A Diuretics to lower systemic vascular resistance (SVR) B Morphine for peripheral dilation and to decrease anxiety C Nitroglycerin to decrease preload and afterload D Dopamine to decrease contractility of the heart E Nesiritide to decrease pulmonary artery occlusion pressure and dyspnea

B C E (EVOLVE) Morphine, nitroglycerine, and nesiritide are all used to treat patients in systolic heart failure. Diuretics will decrease preload, not systemic vascular reistance (SVR). Dopamine will increase myocardial contractility.Cognitive Level: AnalyzingNursing Process: ImplementationClient Need: Physiological Integrity: Pharmacological and Parenteral Therapies

A patient with a family history of coronary artery disease (CAD) has the following laboratory results: total cholesterol, 250 mg/dL; high-density lipoprotein, 35 mg/dL; low-density lipoprotein, 160 mg/dL; and triglycerides, 240 mg/dL. Which interventions should the nurse anticipate? Select all that apply. A Document the normal findings. B Instruct the patient to increase exercise to 30 minutes a day, 5 days a week. C Educate on increasing saturated fat and decreasing fiber in the diet. D Monitor and control blood pressure. E Enroll in smoking cessation classes.

B D E The patient with elevated lipids and a history of coronary artery disease (CAD) should be instructed to increase exercise, monitor blood pressure, and stop smoking. Documenting the findings as normal would be inappropriate because the laboratory test results are not normal. The patient should be educated to decrease saturated fats and increase fiber.Cognitive Level: EvaluatingNursing Process: InterventionsClient Need: Health Promotion and Maintenance (EVOLVE)

A patient is scheduled to have a permanent pacemaker placed for cardiac resynchronization (CRT) therapy. What is the goal of these therapy? Prevent ventricular dysrhythmias. Improve coronary blood flow. Decrease heart failure symptoms. Minimize risk of myocardial infarction.

C A number of clinical trials have shown that cardiac resynchronization therapy (CRT) improves symptoms, functional status, and mortality in patients with moderate to advanced heart failure. New research indicates that CRT may also be beneficial in preventing the progression of heart failure in less symptomatic patients. (EVOLVE)

A patient is admitted with fever, hematuria, and new onset of a cardiac murmur. The patient has a history of intravenous drug abuse and complains of tender spots on the pads of her fingers. She has a low-grade fever, and the nurse notes an enlarged spleen on physical examination. What is the priority nursing diagnosis? Risk for infection related to invasive procedures Risk for anxiety related to lack of availability of narcotics Decreased cardiac output related to alteration in contractility Knowledge deficit related to discharge plans

C Because the patient is experiencing endocarditis, the most important nursing diagnosis is decreased cardiac output related to alteration in contractility. Infection and anxiety are only potential problems, and although knowledge deficit is important, it is not the priority on admission.Cognitive Level: EvaluatingNursing Process: PlanningClient Need: Physiological Integrity: Physiological Adaptation (EVOLVE)

A physiologic cardiac shunt occurs when there is mixing of deoxygenated blood (usually venous blood with reduced oxygen content) with arterial oxygenated blood. In the heart, this is demonstrated by which mechanism? A Atrial septal defect B Patent foramen ovale C Thebesian veins D Ventricular septal defect

C (EVOLVE)

The nurse auscultates a murmur at the fifth intercostal space at the midclavicular line on the left side. The nurse realizes that this is caused by which incompetent heart valve? A Aortic B Pulmonic C Mitral D Tricuspid

C (EVOLVE)

Which question is appropriate for the nurse to ask to determine the quality of the patient's chest pain? A "How long does it last?" B "How severe is it?" C "What is it like?" D "When did it begin?"

C (EVOLVE)

To perfuse the coronary arteries, a patient needs a mean arterial pressure (MAP) of at least 60 mm Hg. Which blood pressure reading will not provide an adequate MAP? A 120/70 mm Hg B 110/50 mm Hg C 90/40 mm Hg D 120/90 mm Hg

C (EVOLVE) 90/40 mm Hg equals a mean arterial pressure (MAP) of 90 + 80/3 = 57, which is inadequate for this patient. The other options are adequate as follows: 110/50 mm Hg = MAP of 110 + 100/3 = 70; 120/70 mm Hg = MAP of 120 + 140/3 = 87; and 120/60 mm Hg = MAP of 120 + 120/3 = 80.Cognitive Level: AnalyzingNursing Process: AssessmentClient Need: Physiological Integrity: Reduction of Risk Potential

Which finding is a clinical manifestation of left-sided heart failure? Increased cardiac output Decreased systemic vascular resistance Cool, pale extremities Vasodilatation of the arterial bed

C (EVOLVE) Failure of the left ventricle is defined as a disturbance of the contractile function of the left ventricle, resulting in a low cardiac output state. This leads to vasoconstriction of the arterial bed that raises systemic vascular resistance, a condition also described as "high afterload," and creates congestion and edema in the pulmonary circulation and alveoli. Patients presenting with left ventricular failure have one of the following: (1) decreased exercise tolerance, (2) fluid retention, or (3) discovery during examination of noncardiac problems. Clinical manifestations of left ventricular failure include decreased peripheral perfusion with weak or diminished pulses; cool, pale extremities; and in later stages, peripheral cyanosis.

In the early stages of pulmonary edema, which arterial blood gas pH would the nurse expect to find? pH of 7.38 pH of 7.34 pH of 7.50 pH of 7.26

C (EVOLVE) In the early stage of pulmonary edema, respiratory alkalosis (pH > 7.45) may be present because of hyperventilation, which eliminates carbon dioxide. As the pulmonary edema progresses and gas exchange becomes impaired, acidosis (pH <7.35) and hypoxemia ensue. A chest radiograph usually confirms an enlarged cardiac silhouette, pulmonary venous congestion, and interstitial edema. Generally the nurse would not expect to see a normal pH (pH between 7.35 and 7.45).

What action should be performed prior to insertion of an arterial line? A Check the patient's platelets for indication of heparin-induced thrombocytopenia. B Obtain a 100-mL bag of 0.9% saline for the flush system. C Perform an Allen test to confirm adequate arterial blood flow. D Set up a double transducer tubing system.

C (EVOLVE) The major advantage of the radial artery is the supply of collateral circulation to the hand provided by the ulnar artery through the palmar arch in most people. Before radial artery cannulation, collateral circulation must be assessed by using Doppler flow or by the modified Allen test according to institutional protocol. In the Allen test, the radial and ulnar arteries are compressed simultaneously. The patient is asked to clench and unclench the hand until it blanches. One of the arteries is then released, and the hand should immediately flush from that side. The same procedure is repeated for the remaining artery.

The nurse is assessing a patient on complete bed rest. Which site should be checked for dependent edema? A The arms B The ankles C The sacrum D The lower legs

C (EVOLVE) The sacrum is the best place to assess the patient on bed rest for dependent edema. In the ambulatory patient, the ankles and the lower legs would not be appropriate. Edema may be present in the arms, but it would not be dependent edema.

The nurse caring for a patient with a temporary transvenous pacemaker notes on the monitor a pacing spike (artifact) that is not followed by a QRS. Which action should the nurse take first? Notify the physician. Decrease the mA. Reposition the patient on the left side. Evaluate the sensitivity threshold.

C The patient has developed failure to capture, and the nurse should place the patient on the left side and increase the mA. Failure to capture occurs as a result of displacement of the pacing electrode or an increase in pacing threshold. Decreasing the mA will make the problem worse. Adjusting the sensitivity will not affect the pacemaker's ability to capture. Notifying the physician is important but should occur after repositioning and assessing the patient.Cognitive Level: EvaluatingNursing Process: ImplementationClient Need: Physiological Integrity: Reduction of Risk Potential (EVOLVE)

When a papillary muscle in the left ventricle ruptures, the mitral valve leaflets do not close completely, resulting in which condition? A Acute myocardial infarction B Aortic valve failure C Cardiac murmur D Systemic venous congestion

C (EVOLVE)

A patient with an arterial line has a systolic blood pressure of 110 mm Hg on inspiration but 128 mm Hg on expiration. This is known as: A pulsus alternans. B narrowed pulse pressure. C pulsus paradoxus. D vascular bruit.

C (EVOLVE) The patient is experiencing pulsus paradoxus, which is a drop in systolic blood pressure owing to increased intrathoracic pressure during inspiration. This is often the result of volume depletion or cardiac tamponade. Pulsus alternans is a pattern of amplitude change noted in end-stage left ventricular failure. Narrowed pulse pressure is a compensatory mechanism noted with vasoconstriction, and vascular bruit is a sound noted when there is vascular turbulence, especially with increased plaque buildup.Cognitive Level: AnalyzingNursing Process: AssessmentClient Need: Physiological Integrity: Reduction of Risk Potential

A patient has been feeling weak for 4 days with a rapid, irregular heart rate. The monitor shows atrial fibrillation with a rapid ventricular response. What is the priority nursing intervention? A Preparing for elective cardioversion B Administering adenosine IV C Performing carotid massage D Initiating an ordered amiodarone drip

D (EVOLVE) The patient is in atrial fibrillation, which has probably been going on for more than 48 hours. The American Heart Association does not recommend cardioversion until the patient has been anticoagulated. The appropriate intervention is to control the rate with a medication such as amiodarone while initiating anticoagulation. Elective cardioversion and adenosine IV would not be appropriate because cardioversion (electrical or chemical) would increase the risk of stroke. Carotid massage is not a nursing function and could cause an embolism.Cognitive Level: EvaluatingNursing Process: InterventionClient Need: Physiological Integrity: Physiological Adaptation

Education for a patient with chronic heart failure would include which information? The need for accurate daily weights Appropriate diet choices Signs of negative changes in cardiac symptoms and when to notify the physician All of the above

D (EVOLVE) Education for a patient with acute or chronic heart failure caused by valvular dysfunction includes (1) information related to diet, (2) fluid restrictions, (3) the actions and side effects of heart failure medications, (4) the need for prophylactic antibiotics before undergoing any invasive procedures, and (5) when to call a health care provider to report a negative change in cardiac symptoms.

A patient with a history of aortic stenosis is admitted for surgery to repair the valve. During cardiac assessment, the nurse would expect to hear which murmur? A high-pitched diastolic murmur The murmur of aortic stenosis occurs during systole. It is auscultated at the aortic area (second intercostal space, right sternal border). Aortic stenosis produces a low-pitched murmur that does not radiate. A A high-pitched diastolic murmur B A high-pitched systolic murmur C A low-pitched diastolic murmur D A low-pitched systolic murmur

D (EVOLVE) The murmur of aortic stenosis occurs during systole. It is auscultated at the aortic area (second intercostal space, right sternal border). Aortic stenosis produces a low-pitched murmur that does not radiate.

The nurse would assess for symptoms of a pneumothorax after placement of a central line into which vein? A Internal jugular B External jugular C Femoral D Subclavian

D (EVOLVE) The subclavian vein is more difficult to access and carries a higher risk of iatrogenic pneumothorax or hemothorax.

A patient is transferred from the medical-surgical unit to the intensive care unit with an acute anterolateral myocardial infarction (MI). Fibrinolytic therapy has been ordered to dissolve the thrombus. What is considered a reliable indicator that fibrinolytic therapy has been successful? Gradual decrease in chest pain Return of creatinine kinase to normal levels Absence of premature ventricular contractions Decrease in ST elevations in affected leads

D When fibrinolytic therapy is successful, ST elevations rapidly resolve. ST elevations indicate myocardial injury; reperfusion restores blood flow and prevents injury. For the same reason, chest pain, which is caused by inadequate oxygen supply to myocardial tissue, is suddenly, not gradually, relieved. Creatinine kinase rapidly increases in reperfusion, a phenomenon known as "washout." Premature ventricular contractions, nonsustained ventricular tachycardia, and other dysrhythmias increase. These are known as (EVOLVE)

The time from the beginning of the cardiac action potential (AP) until the time when the fiber can accept another AP is known as which period? A Excitability B Depolarization C Relative refractory D Absolute refractory

D (EVOLVE)

The nurse is assessing a patient's lower extremities. Which condition is consistent with arterial disease? A Painless, pink fluid drains from an ulceration just above the right ankle. B The feet become cyanotic when the patient sits in a chair. C The nail beds are normal with a capillary refill time of 2 seconds. D Varicose veins are noted on both legs. E Skin on the leg is thin and shiny with a painful ulceration surrounded by eschar.

D (EVOLVE) Skin on the leg that is thin and shiny with a painful ulceration surrounded by eschar is characteristic of an arterial disease of the lower extremity. Painless, pink fluid draining from an ulceration just above the right ankle, the feet becoming cyanotic when the patient sits in a chair, and varicose veins noted on both legs are consistent with a venous disease. Nail beds with a capillary refill time of 2 seconds is a normal finding.Cognitive Level: EvaluatingNursing Process: AssessmentClient Need: Physiological Integrity: Reduction of Risk Potential

A patient has a temporary pacemaker that is rate responsive. The nurse would expect the first three letters of the pacemaker code to be: AOO. DOO. VVI. DDD.

D Rate-responsive pacing is accomplished with a DDD. Both chambers are paced, triggered, and inhibited in this mode. AOO only paces the atrium with no sensing. DOO paces both chambers, but there is no sensing. VVI is a ventricular pacer that senses ventricular activity and inhibits the response of the pacer.Cognitive Level: AnalyzingNursing Process: AssessmentClient Need: Physiological Integrity: Physiological Adaptation (EVOLVE)


Ensembles d'études connexes

Visual Studio Code Key shortcuts - Good Copy

View Set

Chap 58 Chronic Neurologic Problems

View Set

PSY285 ch6 Learning Check Questions + exercises (FINISH EXERCISES)

View Set

EXAM 3-Principles of Radiographic Imaging

View Set

Maryland life and health insurance

View Set

bstrandable NCLEX Miscellaneous 9

View Set

solving quadratic equations: zero product property

View Set

Sociology Final: Chapter 8, Race and Ethnicity

View Set